Jump to content

Wikipedia:Reference desk/Science: Difference between revisions

From Wikipedia, the free encyclopedia
Content deleted Content added
Line 670: Line 670:
Est-ce que c'est une plaisanterie? What noise? And I guess the translator got "potteries" wrong. --[[User:Lisa4edit|Lisa4edit]] ([[User talk:Lisa4edit|talk]]) 21:02, 27 April 2008 (UTC)
Est-ce que c'est une plaisanterie? What noise? And I guess the translator got "potteries" wrong. --[[User:Lisa4edit|Lisa4edit]] ([[User talk:Lisa4edit|talk]]) 21:02, 27 April 2008 (UTC)
:Well, there was that [[X-Files]] episode where Jesus' command to [[Lazarus]] got recorded on a pot being spun in the vicinity, and when it was played as a record, it brought dead people back to life (or at least undeath, which seems a bit odd). --[[User:Trovatore|Trovatore]] ([[User talk:Trovatore|talk]]) 21:08, 27 April 2008 (UTC)
:Well, there was that [[X-Files]] episode where Jesus' command to [[Lazarus]] got recorded on a pot being spun in the vicinity, and when it was played as a record, it brought dead people back to life (or at least undeath, which seems a bit odd). --[[User:Trovatore|Trovatore]] ([[User talk:Trovatore|talk]]) 21:08, 27 April 2008 (UTC)

::C'est "poteries", pas "potteries". Trovatore: Pourquoi penses-tu que c'est une plaisanterie?

::Est-ce que l'épisode est réaliste? Merci. --[[Special:Contributions/99.237.101.48|99.237.101.48]] ([[User talk:99.237.101.48|talk]]) 21:41, 27 April 2008 (UTC)


== A drug called APO 300 ==
== A drug called APO 300 ==

Revision as of 21:41, 27 April 2008

Welcome to the science section
of the Wikipedia reference desk.
Select a section:
Want a faster answer?

Main page: Help searching Wikipedia

   

How can I get my question answered?

  • Select the section of the desk that best fits the general topic of your question (see the navigation column to the right).
  • Post your question to only one section, providing a short header that gives the topic of your question.
  • Type '~~~~' (that is, four tilde characters) at the end – this signs and dates your contribution so we know who wrote what and when.
  • Don't post personal contact information – it will be removed. Any answers will be provided here.
  • Please be as specific as possible, and include all relevant context – the usefulness of answers may depend on the context.
  • Note:
    • We don't answer (and may remove) questions that require medical diagnosis or legal advice.
    • We don't answer requests for opinions, predictions or debate.
    • We don't do your homework for you, though we'll help you past the stuck point.
    • We don't conduct original research or provide a free source of ideas, but we'll help you find information you need.



How do I answer a question?

Main page: Wikipedia:Reference desk/Guidelines

  • The best answers address the question directly, and back up facts with wikilinks and links to sources. Do not edit others' comments and do not give any medical or legal advice.
See also:


April 21

Age of Earth

Ok a friend of mine went to the creation museum and is telling me they presented scientific evidence that earth was only a few thousand years old. I dont buy it. But i thought i should ask, is there any cold hard scientific evidence with out religious rhetoric tied in with it that earth is only a few thousand years old?

No, there is not. None at all. In fact, several different sources have shown consistently that the Earth is about 4.5 billion years old. See Age of Earth for lots of information. See the external link given for more about the "evidence" Young-Earth creationists are using. — Kieff | Talk 01:26, 21 April 2008 (UTC)[reply]
Just to prepare you for any creationist arguments, there was a controversy of pointing to polonium halos as evidence for a young Earth (see Creation geophysics). You can also look at this article. — Ƶ§œš¹ [aɪm ˈfɻɛ̃ⁿdˡi] 02:07, 21 April 2008 (UTC)[reply]
Also read this article. There ought to be a law against it.--Shantavira|feed me 07:59, 21 April 2008 (UTC)[reply]
They do have a commandment, though: "You shall not bear false witness against your neighbor". — Kieff | Talk 08:28, 21 April 2008 (UTC)[reply]
We have a nicely balanced article on Young Earth creationism. Problem that Young Earth creationists face is that not only do they have to find evidence that could point to a young Earth, but they also have to find an alternative explanation for all the other strands of evidence that point to a 4.5 billion year old Earth. They sometimes use the argument that God created the Earth with the appearance of being billions of years old to test the faithful, but this creates two problems. First problem is it this argument depends on God being intentionally deceitful, which is hard to explain. Second problem is that the claimed evidence for a young Earth then shows that God's forgery of an old Earth was imperfect - which is also hard to explain. Gandalf61 (talk) 09:48, 21 April 2008 (UTC)[reply]
A third problem is that forgery of evidence would imply that God has frat boy-philosopher tendencies and wants us to keep wondering if we're brains in vats. --69.134.124.30 (talk) 09:57, 21 April 2008 (UTC)[reply]
See also Donovan's Brain. Edison (talk) 18:53, 21 April 2008 (UTC)[reply]
If they go with the idea that the Earth was created 6,000 years ago with all that carefully-constructed ancient stuff there just to decieve us, then make the claim that the Earth was actually created 10 seconds ago. Ask them to disprove your "ten second" theory. Then suggest that Occam's razor would suggest that the simplest explanation (among all these competing "6,000 years" v. "10 seconds!" theories) is that the Earth is actually as old as it seems: 4.5 billion years. You may not convince them, but you'll have fun along the way.
Atlant (talk) 12:50, 21 April 2008 (UTC)[reply]
  • On this point see Omphalos (theology). --Anon, more than 10 seconds old (as far as I know), 22:03 UTC, April 21, 2008.
Much good fun (as it says in escaping from fundamentalism by james barr). Bear in mind though that the Bible describes Adam created as an adult man not a new-born baby and therefore "created old" isn't too much of a stretch. --BozMo talk 12:56, 21 April 2008 (UTC)[reply]
Not wanting to start a debate here or go OT but the God you're referring to is the same one who brutally killed 'sinners' (Noah's Ark, Sodom and Gomorrah.....), asked a father to murder his son for no good reason, used acts which would likely be considered terrorism in this day and age including biological warfare (Plagues of Egypt) etc etc; I don't think 'intentionally deceitful, which is hard to explain' is hard to explain in the eyes who believe everything in the bible should be taken literally Nil Einne (talk) 19:09, 21 April 2008 (UTC)[reply]

Alright folks let's keep this clean of ranting against Christianity shall we? ;) Regards, CycloneNimrodTalk? 19:33, 21 April 2008 (UTC)[reply]

Just to be clear, I wasn't ranting against Christianity, simply pointing out that if you take the bible literally, then you have far bigger issues to explain about God then him/her being 'intentionally deceitful'. Note that there are a lot of Christians who don't interpret the bible completely literally and many explanations Christians come up with for the varying complexities that arise if you intepret the bible literally which obviously don't need to be and shouldn't be discussed here. Whatever Christians want to believe, is up to them, and this obviously isn't a forum for such a discussion. Nil Einne (talk) 22:01, 21 April 2008 (UTC)[reply]

Just ignore "scientific creationism" -- it's the worst kind of oxymoron. —Steve Summit (talk) 00:31, 22 April 2008 (UTC)[reply]

There are those of certain sects that believe that the Earth must be very young in order to conform with received scripture. From that point of view they then look for evidence for it and try to find ways to discount evidence against it. It is a limited view of the scripture, and it is a view which has no scientific validity either in content or in methodology. --Captain Ref Desk (talk) 14:55, 22 April 2008 (UTC)[reply]

Energy and momentum

It is a general rule that in a collision, unless otherwise stated, momentum is conserved while energy is not. Now I can understand why energy may not be conserved (i.e. it gets transfered as heat), but why is momentum assumed to be conserved? After all, when objects collide, temperature (i.e. the motion of atoms) increase, and thus their momentum. On another point, in an ideal collision, when energy and momentum are both considered to be conserved, if an object hits another object at rest, all momentum is transfered to the second body. Now I understand the math behind it, but how can this be explained intuitively?

Are you talking about particles or objects? In a collision with objects, heat isn't the only way energy is lost (friction, elasticity etc). In an object, temperature is not related to the overall momentum. Momentum can only be assumed to be conserved in a closed system, because momentum in a collision is conserved (m1v1+m2v2=m1v1+m2v2). Momentum is related to velocity, thus energy.
In an ideal collision, no energy is lost, so it all must be conserved. Note that all momentum will only be transferred if the two objects have the same mass.

Luxosus (talk) 02:32, 21 April 2008 (UTC)[reply]

Momentum is conserved even if heat is produced because the thermal movement of particles is random with respect to direction (more specifically a Maxwell distribution) and thus the momenta cancel each other. If they don't cancel - well, then the object is actually moving in a certain direction which contributes to its macroscopic momentum. Icek (talk) 03:12, 21 April 2008 (UTC)[reply]
How is energy lost due to elasticity?
Internal stress due to compression\expansion. He didn't mean it in the context of elastic collisions. — Kieff | Talk 04:13, 21 April 2008 (UTC)[reply]
Both are conserved. However, energy has many different modes: overall linear and rotational kinetic energy, potential energy of the objects' chemical bonds (e.g. deformation), temperature, etc. It is very difficult to predict or measure how much energy is transferred to each different mode. Momentum, on the other hand, is awfully simple under the circumstances, scales, and velocities in which we usually pose such problems: mass is constant (and usually not exchanged significantly between the objects undergoing collision), so the only unknowns are the velocities of each of the objects involved, and there you can even analyze three directions independently. So it is convenient to work directly with momentum, and not so convenient to work with (kinetic) energy except in a secondary fashion. --Prestidigitator (talk) 05:04, 21 April 2008 (UTC)[reply]
A deeper (and therefore possibly less useful) answer to the "why" question is that conserved quantites are associated with symmetries of physical systems and laws - see Noether's theorem. Linear momentum is conserved because the laws of physics are invariant with respect to translations in space; if we move everything a metre to the left (or, equivalently, move our origin a metre to the right), we don't expect the laws of physics to change. Similarly, conservation of energy is a consequence of invariance with respect to translations in time, and conservation of angular momentum is a consequence of invariance with respect to rotations. Gandalf61 (talk) 09:21, 21 April 2008 (UTC)[reply]

Electrodynamics application

Hello,

I've found these very informative electrodynamics simulations applets : http://www.falstad.com/emwave1/ and http://www.falstad.com/emwave2/ .

Does anyone know of a similar program (not necessarily an applet, maybe a downloadable application) which does essentially the same thing but in 3D ?

I would like it if it simulated Maxwell's equations totally (maybe even with magnetic charge to see what it would be like, but I suppose that would be quite unlikely).

Thanks. -- Xedi (talk) 07:40, 21 April 2008 (UTC)[reply]

Cracked Walls

What causes cement paste cracked on walls and solutions to prevent? —Preceding unsigned comment added by Tajteddy (talkcontribs) 07:51, 21 April 2008

Do you mean cracking of the interior plaster, or cracking of the concrete structure of the wall itself?
The interior plaster usually cracks due to drying in the first few months after it is applied to the wall. I don't believe much can be done about it, except to wait several months and then fill the crack with a commercially available filler. After filling the cracks, the wall can be painted or wallpapered to cover up the repair.
If the concrete structure of the wall is cracked, it could be due to settlement or subsidence, and is much more serious - the wall could fall down - so contact a professional builder.
Astronaut (talk) 11:32, 21 April 2008 (UTC)[reply]

Try adding a synthetic resin or plastic dispersion to the mix. NB: This may not work on your particular surface and makes the plaster way more expensive! Worst case scenario you might end up with big sheets of dried plaster falling off the surface underneath. There are lots of factors contributing to cracks. A few that come to mind are: shrinkage due to (uneven) water loss in the hardening phase, water absorption and subsequent drying of the surface underneath, temperature gradients during hardening, imperfect bonding to the surface underneath etc. etc. Lisa4edit--Lisa4edit (talk) 06:41, 22 April 2008 (UTC)[reply]

eavesdropping 2G/2.5G voice calls

How can wiretapping voice calls on 2G network be done? Suppose both the users are under the same cell. In that case, what kind of electronic circuitry would be required to tune into all the carrier frequency channels, demultiplex them to the 8 channels, and record? Even after the demuxing, how can we extract the specific audio data and decode it? I am an Electronics engg. 2nd yr student. It would be quite useful for me in case you can provide me with detailed info on the circuitry,... Bobatnet (talk) 10:54, 21 April 2008 (UTC)[reply]

Given encryption is AFAIK used by the majority of mobile providers out there, this is probably very, very difficult and costly without the cooperation of the telephone company. Your only reasonably (but still probably rather expensive and difficult to set up) option would be to use something like an IMSI-catcher as mentioned in Telephone tapping. Note that the use of one or for that matter, any form of tapping could very well be illegal in your jurisdiction, I suggest you speak to your supervisor or some other person at university before trying anything. Nil Einne (talk) 18:56, 21 April 2008 (UTC)[reply]

Quantum devices

I know it is not possible with current technology, but would it be concieveable to use quantum entanglement to construct a nano-scale remote sensing device? Astronaut (talk) 11:38, 21 April 2008 (UTC)[reply]

As I understand it the answer is yes, but no useful information can be transmitted in that way. If you could read the state of an individual particle, you can then determine the state of it's entangled particle, even at a great distance. However, changing the state of one particle does not change the state of the other, which is what would be necessary for communication. StuRat (talk) 16:25, 21 April 2008 (UTC)[reply]

Invertase

What is invertase and why would a candy manufacturer put it in a candy? For example, it is in the ingredient list for 'Cherry Blossom'. Thanks, Wanderer57 (talk) 11:44, 21 April 2008 (UTC)[reply]

See invertase. --antilivedT | C | G 12:02, 21 April 2008 (UTC)[reply]

Diagrams for scientific journal articles

How are diagrams of gene promoters (particularly with many, many lines showing the positions of CpGs within a CpG island) and the contents of genetic plasmids etc constructed? Special software? Some scientists blatantly try to make do with PowerPoint but I don't think that's a very effective approach. ----Seans Potato Business 15:22, 21 April 2008 (UTC)[reply]

There's lots of specialized software for drawing biochemical structures. The biggie is probably the ChemOffice suite. DMacks (talk) 17:04, 21 April 2008 (UTC)[reply]

Rate of neutrino oscillations

At what rate, in terms of Hz, say, do neutrinos oscillate? Or am I misunderstanding the issue? The article is a bit too technical for me.--Fangz (talk) 16:56, 21 April 2008 (UTC)[reply]

This page (sorry it's not a wiki page) may clarify things a bit more http://www.ps.uci.edu/~superk/nuosc.html Hope this helps. Lisa4edit —Preceding unsigned comment added by Lisa4edit (talkcontribs) 06:11, 22 April 2008 (UTC) --Lisa4edit (talk) 06:11, 22 April 2008 (UTC) oops I forgot.[reply]

Do orbiting satellites have shadows?

topic. Bellum et Pax (talk) 17:39, 21 April 2008 (UTC)[reply]

Yes, just very small and likely obscured by all the other (diffuse) light present during daylight. They also move very quickly. Compare with high-flying airplanes which have shadows that are visible on the clouds below by their occupants but are likely unnoticeable by those on the ground. -- Flyguy649 talk 17:43, 21 April 2008 (UTC)[reply]
Um, well, sort of. The thing is, when an artificial satellite transits in front of the Sun, as seen from the Earth, it never occludes the entire disk -- it's just a little speck passing in front of it. Therefore it has no "shadow" in the sense of something with a sharp boundary. In eclipse terminology, it has a penumbra but no umbra. --Trovatore (talk) 18:40, 21 April 2008 (UTC)[reply]
At any point on the ground, the angular subtense (image diameter) of the sun would be larger than that of an artificial satellite. Therefore a satellite would have no Umbra or spot on the ground where there was no view of the sun; instead for an instant you might see the satellite sillhouetted against the sun. So no, there would not really be a shadow. Unless there is a really big satellite up there, say 4500 feet diameter at an altitude of 100 miles, or 2.5 miles diameter at 300 miles (please check the math). Edison (talk) 18:49, 21 April 2008 (UTC)[reply]
Quite right. Here's a stunning photograph of the International Space Station and the Space Shuttle transiting the sun. Even though they are large artificial satellites, they are very small in angular size compared to the disk of the sun. -- Coneslayer (talk) 18:54, 21 April 2008 (UTC)[reply]

Romantic implications of garlic

When two equivalent persons consume an equivalent quantity of garlic, do they both a) detect the garlic exuding from the other person or b) detect no garlic or c) other? ----Seans Potato Business 19:06, 21 April 2008 (UTC)[reply]

Well since you can usually detect garlic exuding from yourself, I would have to go with the first one Nil Einne (talk) 19:12, 21 April 2008 (UTC)[reply]
When both people are exhaling and have their mouths open, though, I would expect one's own smell of garlic to mask the other person's smell. Let's suppose the distance from the nose to the mouth is 10 cm, and the distance form the nose to person B's mouth is 50 cm. Assuming smell dilutes with the square of the distance, person B's smell would be 4% one's own, which is not easily detectable. --Bowlhover (talk) 20:17, 21 April 2008 (UTC)[reply]
Would you be as kind as to upload a picture of yourself on your user page? :) This mouth to nose distance of yours has me rather curious. 81.93.102.185 (talk) 20:38, 21 April 2008 (UTC)[reply]
Well, if I used the reasonable 5 cm, that would make Person B's relative smell even weaker. But sure, I'll give you a picture: [1]. --Bowlhover (talk) 03:48, 22 April 2008 (UTC)[reply]

The distance theory would work if there wasn't the fact that repeated sensory stimulus leads to heightened tolerance or more precisely diminished response. Thus difference between s.o. else's oder and one's own should be perceived more acutely. (Also see Action potential) Lisa4edit --Lisa4edit (talk) 05:36, 22 April 2008 (UTC)[reply]

Nudity addiction

I am sory if I am asking for an advice but it is urgent. I have an addiction where I like people being naked especially women and there is an free option where we can do sex on any woman or touch a single part of woman's body. the problem is as a Muslim and a Muslim cannot see a person being in a naked state because it is considered as a sin. I need your help on: how to get rid of this addiction before it is too late and when I mean that, I mean I will eventually become as a nudist. QASAP. Thank you. sorry if I used a spiritual sense in it. —Preceding unsigned comment added by 192.30.202.20 (talk) 22:26, 21 April 2008 (UTC)[reply]

Muslims can see nude women. How do you think they reproduce? 216.120.213.165 (talk) 22:31, 21 April 2008 (UTC)[reply]
I believe it's ok in marriage. -mattbuck (Talk) 23:12, 21 April 2008 (UTC)[reply]
I seem to recall that Saint Paul wrote in Corinthians, "It is better to marry than to burn." If you are a Muslim male, marriage might be a solution to your desire to see and touch a naked female. Edison (talk) 00:12, 22 April 2008 (UTC)[reply]
There are a number of belief systems (e.g. New Age philosophy) which would encourage you to embrace your addiction. You can abandon Mohammed and run to the mountain. - Kittybrewster 17:06, 22 April 2008 (UTC)[reply]
If you are legitimately concerned about addiciton and behaviors that you are having difficulty controlling/resisting, then you should speak to a doctor or mental health professional about your problem. The Reference Desk is not equipped to address physical or mental health problems. Dragons flight (talk) 01:00, 22 April 2008 (UTC)[reply]
I'd hardly call it an addiction, and more of a basic fact of human nature. Especially in societies where nudity is taboo, which only lends an air of mystery and "naughtiness" to it. Basically, by forcing people to cover up, we put even more emphasis on sex. Only we add to the negative pressure to avoid this natural function of our bodies, which leads to exactly the crisis you find yourself in. Humans are kinda messed up that way! -- Kesh (talk) 00:17, 23 April 2008 (UTC)[reply]

Octopi die after reproducing - why?

Our octopus article states that female octopi die soon after their eggs have hatched, explaining that endocrine secretions are the cause - but it doesn't explain *why* the octopi face such a 'genetically-programmed death' as a matter of course. What is the benefit to an octopus of reproducing on only one occasion in its lifetime? Is there even a 'benefit', as such in evolutionary terms? --Kurt Shaped Box (talk) 22:32, 21 April 2008 (UTC)[reply]

I don't know of an "on one occasion" benefit, but in general terms, there's no particular evolutionary harm in things that kill you exclusively after you've reproduced -- which is, of course, why old age happens at all. You could say it's only a matter of degree between an octopus mother dying 60 minutes after giving birth, and a human mother dying 60 years later. —Steve Summit (talk) 00:26, 22 April 2008 (UTC)[reply]
On the other hand, a human female's system does not actively 'commit suicide' directly after giving birth. I thought that it was a basic 'rule' that all living things have an instinctual desire to survive and to reproduce as often as possible - this 'chemical suicide' thing seems to run completely contrary to that in my mind. Sure, mama octopus might lay thousands of eggs in a single clutch, so there's a fairly good chance that her genes will survive - but why not live to do it all again the next year to make doubly sure (baby octopus life is fairly cheap as I understand it)? I'm struggling to understand how and why these 'suicide glands' even evolved in the first place. --Kurt Shaped Box (talk) 01:01, 22 April 2008 (UTC)[reply]
There are any number of possible reasons why parents might be programmed to die, such as:
  • To prevent mama from eating the babies
  • To free up resources (e.g. food) for the babies
  • To eliminate the possibility of mother/child incest
  • So the babies can eat the mother
  • Because the babies will attract fewer predators on their own
  • Because the mama no longer has viable eggs (in mammals, the progenitor cells for all viable eggs are created when the girl is still a fetus, and these cannot be replaced later on. It's possible that the mama octopus uses all her eggs in one go and similarly was never capable of making more as an adult.)
  • Because the act of producing a clutch damages the mama in a way that it cannot recover from.
I don't know if any of these are correct for octopi, but it is certainly possible to imagine a wide range of potentially evolutionary incentives to not having an organism continue past reproduction. Dragons flight (talk) 01:21, 22 April 2008 (UTC)[reply]


Just by the way, octopi is generally considered substandard usage. Octopus is of Greek rather than Latin origin, and the Greek plural would be octopodes (pronounced roughly ok-TOP-oh-deez). That plural is occasionally used in English, but the accepted English plural is octopuses. --Trovatore (talk) 02:28, 22 April 2008 (UTC)[reply]

"Octopi" has been used so much that the substandard usage has been blurred. bibliomaniac15 Do I have your trust? 02:35, 22 April 2008 (UTC)[reply]
Here's an article that might be relevant: Evolution of ageing. --Allen (talk) 04:08, 22 April 2008 (UTC)[reply]
Aww, man! Next you'll be telling me that hippopotami (another one I've always used) is 'substandard' too... :( --Kurt Shaped Box (talk) 23:59, 22 April 2008 (UTC)[reply]

There are many different reproductive strategies (many different ways to skin a cat) as mentioned in our article reproduction. Look up semelparous organisms and see our article r/K selection theory.--Eriastrum (talk) 16:00, 22 April 2008 (UTC)[reply]

Yes, thanks very much for all the answers, folks... :) --Kurt Shaped Box (talk) 23:59, 22 April 2008 (UTC)[reply]

I studied at several well-known college campi, and I had never heard the term "octopodes." The things one learns on Ref Desk! Actually, "The New Shorter Oxford English Dictionary" gives the plurals of "octopus" as, in order, "octopuses," "octopi," and "octopodes." "The American Heritage Dictionary of the English Language" gives the plurals, in order, as "octopuses," octopodes," and "octopi." On neither side of the pond does "octopodes" appear to be the preferred plural. Now I must go feed the mongeese. Edison (talk) 22:57, 23 April 2008 (UTC)[reply]

Speeding Up causes Decompression?

I know the air going over an aircraft's wing becomes 'Accellerated', and because of this, the same air loses pressure and the plane flies.

Is it because of the surface of the plane's wing, or is it a rule that Accellerating air Loses Pressure? P.L. —Preceding unsigned comment added by 72.39.252.217 (talk) 23:00, 21 April 2008 (UTC)[reply]

I believe it's because there is a fixed volume of air going over it per second, and so if it's going faster, there must be a lower pressure. But I hated fluid mechanics, hopefully someone else can answer. -mattbuck (Talk) 23:10, 21 April 2008 (UTC)[reply]
It's the bernoulli principle at work. But as a certain Steve who doesn't edit the RD anymore would often love to explain in obscene detail, angle of attack bears a much greater responsibility for the lift of modern planes. Someguy1221 (talk) 23:48, 21 April 2008 (UTC)[reply]


April 22

different times

can you please tell what the abbreviations A.M.and P.M. stand for as in desiginating morning and night in telling time.I think P.M. stands for prime meridian but not sure.....Thank-You —Preceding unsigned comment added by 75.170.141.107 (talk) 01:41, 22 April 2008 (UTC)[reply]

ante meridiem and post meridiem See our article "12-hour clock". --Milkbreath (talk) 01:48, 22 April 2008 (UTC)[reply]
Additionally, the prime meridian is the line of longitude that passes through Greenwich (i.e. 0 degrees East/West). Angus Lepper(T, C, D) 21:49, 22 April 2008 (UTC)[reply]

electrical phenomenon

I'm looking for a reference which might describe an electrical phenomenon I've just witnessed.

I made a plastic cylinder about an inch wide and about four inches in diameter and wrapped it with about 40 turns of enameled wire and encapsulated it with glue. I hooked up this coil up in place of my stereo speakers and played a tone. Although there is no speaker diaphragm and no magnets I can hear the tone coming from the coil. How is this possible? 71.100.160.42 (talk) 05:57, 22 April 2008 (UTC) [reply]

The skeptic in me says that it isn't happening. How exactly is it hooked up to your speakers? Sure the sound is not coming from the speakerS? What kind of plastic? PVC?--Shniken1 (talk) 06:06, 22 April 2008 (UTC)[reply]
If the wire isn't 100% glued down in every place, then the wire will move in response to the changing magnetic field in the coil and you'll definitely hear that. You might also take a steel food can, cut out one end, and then hold the other end near your energized coil; that should produce quite a bit more sound than the wire alone. Yo might also want to read about magnetostriction although I don't think copper wire itself is susceptible to that. But it is a big factor in why steel cores in electromagnets make noise.
Atlant (talk) 11:53, 22 April 2008 (UTC)[reply]

Please be careful. It sounds like you only have about 10 feet of wire in your coil, which will look to the amplifier like a dead short. This extremely low impedance coil will draw excess current from the amplifier output and could destroy it. To simulate the coil of the speaker you would need many turns of fine wire in a series of layers to get up to 4 ohms (or 8 ohms) depending on the amplifier specifications. Otherwise an audio transformer could be used to step up the coil impedance to a value the amplifier could handle. That said, the windings of the coil could attract and repel each other, allowing some tone to be heard. The first telephones used a steel needle (like a skewer) suspended on a sound board inside a coil to reproduce sound. A coil alone, without something ferromagnetic, would be far less efficient at producing sound. Edison (talk) 14:09, 22 April 2008 (UTC)[reply]

Yes, thanks for the warning. What components would be effected? Would the damage caused by low speaker output resistance resulting in a virtual short circuit cause the MOSFET to be destroyed or only degraded? I have an AM/FM radio tuner, tape and auxiliary amplifier I found at a thrift store that requires an extreme balance knob offset in order for output to be equalized. Also would an 8 ohm audio transformer hooked to the speaker output solve the problem? 71.100.160.42 (talk) 20:27, 22 April 2008 (UTC) [reply]
Your point is well taken, but remember that we're talking about reactive impedance, not DC resistance. And most solid-state amps are pretty good at protecting themselves from short circuits anyway, especially when you're only driving them at moderate "volumes". If it's a real concern, our poster can always put a small fixed resistance (say, two ohms) in series with his test coil.
Atlant (talk) 17:41, 22 April 2008 (UTC)[reply]

If the coil had an impedance of say .2 ohms and you put a protective 2 ohm resistor in series, then the coil would only get about 1/11 of the power and would drop way down in audibility. Seems like a losing proposition. Calculate the resistance of your winding from a wire table, or the reactance from complicated formulas to combine with resistance to get impedance. Then the turns ratio or impedance ratio of the transformer would have to be such as to somewhat match the coil impedance. Perhaps an old transformer from a public address speaker designed for the "70 volt" system would work, or an output speaker from an old vacuum tube audio device. Or you could build am amplifier designed to drive a low impedance load. The impedance of a speaker winding is mostly resistance, isn't it? Someone might try measuring the resistance of a speaker coil labelled 8 ohms. And I don't know that all transistor amps are self protected against shorts, although I know some amps have fuses or breakers. Edison (talk) 22:41, 23 April 2008 (UTC)[reply]

I don't know this for certain, but I believe that most of the "impedance" seen in a speaker winding is, in fact, the counter EMF that is generated as the voice coil moves through the magnetic field. I'm pretty sure if you stuck a speaker on an impedance bridge, you'd get a much different results when the voice coil is free to move as compared to what you'd get if the voice coil is mechanically restrained from moving.
Atlant (talk) 12:44, 24 April 2008 (UTC)[reply]

Magnetism

Hi, Can anyone tell me what actually causes magnetism?? How does a magnet know that a thing near it, is made of iron, nickel, etc.....I mean, why should it attract only ferromagnetic substances? And why do certain substances have strong magnetic properties while others don't ? What's so special about them?? I've read in my physics textbook that if a magnet is divided it forms two new magnets and so on. So, at the smallest level, will an atom act as a magnet? If it does, why doesn't every other atom act as a magnet, ie. , why can't another substance, like calcium or magnesium be magnetic? And also, how does it act from a distance? (I have similar doubts about most other forces like gravity. I've heard of gauge bosons but I can't imagine how if some particles go back or forth it can pull or repel an object.........) 116.68.71.178 (talk) 06:27, 22 April 2008 (UTC) The 15-yr old[reply]

  • No-one can tell you what "causes" magnetism, we can just say it exists and describe its effects.
  • A magnet doesn't "know" anything, it just emanates a magnetic field.
  • A magnetic field does not only attract ferromagnetic substances, it exerts a force on any ferromagnetic, paramagnetic and diamagnetic materials and also on moving electric charges.
  • Strong magnetic properties result in materials where the conditions are right, see Ferromagnetism#Physical_origin, which also describes how a single atom acts as a magnet.
  • As to how it acts from a distance, the best I can do there is: because the electromagnetic force acts at a distance. You can have your doubts about forces acting at a distance, but then don't turn on the radio or jump off buildings ;)
Hope that helps a bit, others can add more. Franamax (talk) 08:06, 22 April 2008 (UTC)[reply]

Thanks a lot. This cleared up most of my doubts. And I certainly won't jump off a building to see the working of forces at a distance! 116.68.70.17 (talk) 12:26, 22 April 2008 (UTC) I asked the question[reply]

Note that gravity might not work the same way as the other forces—this is still an unresolved issue in physics. It might "just" be a deformation in spacetime, and the apparent force might "just" be matter going along a path of least resistance in spacetime.
As for "on the smallest level, will an atom act as a magnet?"—it depends on the atom, but magnetism is basically related to the fact that electrons and protons have different charges, and that under certain configurations the collective charges line up and produce a field (which is why not everything appears magnetized to us). On the smallest level (or something approximating it) a lone electron acts like a magnet.
I too find gauge bosons hard to visualize—clearly thinking of them as little billiard balls that fly out of things is wrong, though it is what comes to mind. Instead just think of them as "the things that cause the fields". The fields are easy enough to visualize abstractly, even if they don't conjure up much by means of a mechanical analogy. --Captain Ref Desk (talk) 13:18, 22 April 2008 (UTC)[reply]
116.68, keep in mind here that you will never get an answer to "exactly why does this happen". Gauge bosons, photons, electrons, whatever - don't get confused by the talk about particles and waves, there are no such things, as CRD says, there are no such little billiard-balls flying about. Our imperfect human minds need analogies we can understand, like "particles" and "waves", because those are things we can see in our physical lives. These ideas help us make predictions about what will happen in our world, but don't embrace the idea that there is really some little thing flying around that says "gauge boson" on the side. See also Tao and The Tao of Physics. Franamax (talk) 00:49, 23 April 2008 (UTC)[reply]

Thanks. That helped, and also opened up another question in my mind.......... How do we (in the sense,humans) know about so many things like bosons, waves,etc. etc. and other so, so minute things? Can't the way we see it be wrong?? I've read in a book that Immanuel Kant (I believe it's him, but not sure) classified everything into two - "nuomena" that's the absolute truth that can't be known , and "phenomena" - truth as we see and experience it. So, everything we perceive could be different from reality, and what we understand could be just within the limitations of our brains senses..........I don't know much but it was just a question that popped up. 116.68.70.86 (talk) 12:25, 23 April 2008 (UTC) The 15- year old who asked the question.[reply]

Well, you're not going to find the ultimate answer to that question here - it's been plaguing people since we noticed we were conscious beings. If you find that question interesting, you'll like philosophy (as you seem to have discovered?). Anyhow, in terms of entities that science has "discovered" like photons, electrons, etc, it is fairly pointless to argue about whether or not they really exist. From a scientific point of view, positing the existence of photons or elections with specific properties allows us to simply explain or predict phenomena that we observe. It's kind of a closed system - we observe something, we come up with the simplest possible explanation, and test to see if that explanation can always explain the phenomenon, and make sure that explanation doesn't contradict any other explanations. So...is it real? It doesn't matter, because it works. --Bmk (talk) 04:52, 25 April 2008 (UTC)[reply]

That's a good point! So, I needn't bother, if all things I see, exist in reality, I just need to mind my own bussiness, in a sense. Thanks. 116.68.70.51 (talk) 10:34, 25 April 2008 (UTC)A 15 year old[reply]

Maclean vs Maclear (OCR typos)

OK, OCR typos aren't really a science question, but could someone look at this webpage (cached version) from the Royal Society, and check that I'm not making things up here. That page lists a "Thomas Maclean" as one of the Royal Medal winners for 1869. I spent a while searching for this guy on Wikipedia and the internet, before realising that this seems to be a typo for Thomas Maclear. A few other scattered webpages refer to him as "Thomas Maclean" (for example, this book from the 1880s), but most (for example, here) say Thomas Maclear and never mention any alternative spelling. Is this likely to be a mistake that spread at some point, or could there be another reason? I've corrected Royal Medal (and left a note on the talk page), but is it worth pointing out that the Royal Society website seems to have the wrong name? Or is it just possible that all the other sources are wrong and that Maclear is the mistake?? Carcharoth (talk) 06:41, 22 April 2008 (UTC)[reply]

Searching the RS archives for Thomas Maclean finds nothing, whereas search for Thomas Maclear finds him listed as a fellow. I'd say the RS has it wrong on the first page you link, but right elsewhere, and Maclear is likely the right name. Franamax (talk) 07:01, 22 April 2008 (UTC)[reply]
And the London Gazette of 1860 awards the knighthood to Maclear. Looks like a typo on the list of medal winners. Franamax (talk) 07:19, 22 April 2008 (UTC)[reply]
Should have thought of that! Thanks. The fellows archive page lists his Royal Medal, just to confirm it. Carcharoth (talk) 07:33, 22 April 2008 (UTC)[reply]
I hit "Contact Us" on the RS website and let them know. We'll have to share the honours I'm sure they'll confer for pointing it out. Hope it's a fruit basket! Franamax (talk) 07:37, 22 April 2008 (UTC)[reply]
What about the book from the 1880s...? Carcharoth (talk) 07:51, 22 April 2008 (UTC)[reply]
According to the site, the transcriptions were made by John and Jerry Grover, and according to the contents page for Peek's Account, it was transcribed from hand-written text, so it is fairly certainly in error. Not an OCR error, a "what the heck does this say" error of the old kind :) Franamax (talk) 08:33, 22 April 2008 (UTC)[reply]
Hedge that, they might mean his "hand" as in a sockpuppet, I'd still say it's a transcription error though. Franamax (talk) 08:40, 22 April 2008 (UTC)[reply]

"Buffer" for speech?

It seems to me that the brain has a sort of "buffer" for speech, analogous to a computer's buffer for keystrokes.

I have noticed, for example, that sometimes if I am not completely paying attention when a person is talking, that this "buffer" will catch a few seconds of their speech and hold it, but my brain will not parse the speech into meaning until I have turned my attention to it.

I do not know if this buffer is for sound in general or if it is just for speech.

What is this buffer called? —Preceding unsigned comment added by 71.139.143.2 (talk) 08:03, 22 April 2008 (UTC)[reply]

The brain has a sensory memory and it is used for any kind of information not just speech. If you hear a bell ringing without paying too much attention you can retrieve the number of rings within seconds. Mr.K. (talk) 09:39, 22 April 2008 (UTC)[reply]
Occasionally, find that I occasionally need to "replay" a string of phonemes in my head and run a sort of PRML algorithm based on the likely context of the phonemes before I can figure out what's being said. (See also Maximum likelihood.) This occurs when the phonemes could match "phrase A" just as easily as they could match "Phrase B". That's when I notice this sort of short-term auditory memory.
Atlant (talk) 12:07, 22 April 2008 (UTC)[reply]
Steven Pinker's The Language Instinct discusses blowing your buffer with sentences that require you to keep things on a stack until the last word:

That many teachers are being laid off in a shortsighted attempt to balance this year's budget at the same time that the governor's cronies and bureaucratic hacks are lining their pockets is appalling.

--Sean 12:37, 22 April 2008 (UTC)[reply]

This has been called "echoic memory." A stream of speech or other sound seems to flow through memory and fade away, but if there is a retrieval cue, we can retrieve several seconds of it. An example is if one is reding the paper at the breakfast table and the significant other is chatting away, or daydreaming in class while a teacher lectures away, the reader or daydreamer can often retrieve enough of the unattended speech to give a response that makes it appear they were paying attention. The Wikipedia article does not cover some earlier experiments in the area which dealt with a stream of unattended speech. Edison (talk) 14:01, 22 April 2008 (UTC)[reply]

The interesting thing is that you can file "false" memories. Your brain (and you) will be convinced that the thing that it remembers was what actually happened. This fact has recently shed a lot of doubt on the validity of eyewitness accounts. --Lisa4edit (talk) 07:36, 23 April 2008 (UTC)[reply]

Since eyewitness accounts differ anyway, wouldn't that have counted against their validity before this fact was applied? Julia Rossi (talk) 09:03, 23 April 2008 (UTC)[reply]
chomsky's theory involves a more serious buffer, which not only holds the content of the speech until the end but after that it is worked on to generate the "deep meaning" so that you can understand the difference between "man eating sharks" and "man eating sardines", for a trivial example; or convert "the dog bit the man" and "the man was bitten by the dog" etc. to identical final products. Gzuckier (talk) 19:09, 23 April 2008 (UTC)[reply]

While watching a typist take dictation a few years ago, I noticed lthat she only started typing a few seconds AFTER the dictating began. Although people said that typists did not care about or understand what was being read to them, I realised that the typist could not type effectively without getting the gist of what was meant, and only then typing the words. I think all typists (I am a pretty fast touch typist mystelf and have often typed from dictation) need to understand what is intended. This can be clearly seen if they are asked to type a list of unrelated words. It is rare for a typist, even one who “doesn’t care” about the significance of what is intended to make mistakes with homonyms, for example writing “the HEART ran through the forest”, when HART is intended. Moreover, all typists are irritated when the dictater does not offer the prose in naturally logical bite size pieces, and stops half way through phrases and so on. In like manner, I believe everyone listening to speech partially disengages mentally from the stream, and then “catches up” after a couple of seconds. The art of being able to easily comprehend long and complex passages of speech or text is much atrophied in contemporary society. Hava a look at writing, either novelistic or argumentative from the 1800s, People were expected to read sentences that were longer than most paragraphs are now, and with the subject separated from its associated verb by many lines of text, including phrases nested within other phrases. Myles325a (talk) 07:14, 24 April 2008 (UTC)[reply]

What are the "doors" or entryways on a ship/boat called?

Specifically, the entryways from one section to another. Not the cabin doors. BrokenSphereMsg me 16:47, 22 April 2008 (UTC)[reply]

Hatch. Hatchway. Companionway. Watertight bulkheads? Other terms?
Atlant (talk) 17:33, 22 April 2008 (UTC)[reply]
The entryway through which the person in the photo is using to go through. She's climbing over a bulkhead but what's the oval doorway called? --BrokenSphereMsg me 17:43, 22 April 2008 (UTC)[reply]
The oval hole? An oval hole at most, I'm afraid. The best alternative is going to be the specific type of door, bulkhead and similar. 81.93.102.185 (talk) 18:56, 22 April 2008 (UTC)[reply]
If there's a watertight door that can be used to shut off the area then I guess a watertight door, if said door is normally kept open to facilitate passage? BrokenSphereMsg me 20:15, 22 April 2008 (UTC)[reply]
How would the hole be called a "door?" Edison (talk) 20:24, 22 April 2008 (UTC)[reply]
If there is a watertight door (unseen here perhaps, I can't recall) that can be shut to cut off sections of a vessel to halt flooding. This pic is of a submarine, after all. However my question is for all maritime vessels. "Door" doesn't seem very nautical to me, however for cabins or quarters I guess those can have "doors" for privacy, right? BrokenSphereMsg me 20:29, 22 April 2008 (UTC)[reply]


Speaking for the US Navy, they are called "Watertight Doors" and a google search for that term gives lots of pictures. The example you show has been modified/mangled to no longer work. You can tell it used to be a WTD because the "coaming" (the wide lip all around the oval) is a good 3/4" thick, and 5 or 6" wide , and there is still one dog-wedge visible on the lower left. When new and in use, there were at least 4, and probably 6 or 8 of these evenly spaced around the coaming. The door would have been on the other side of the frame, with "dogs" (latches) that ran up these wedges, forcing a rubber gasket on the edge of the door up against the coaming. In it's current condition, it's only purpose is to injure people, so it is normally referred to as a "knee-knocker". -SandyJax (talk) 20:33, 22 April 2008 (UTC)[reply]
A company which sells watertight doors (with dogs) for ships calls it [2]an "opening" (more specifically "clear opening." Some patents refer to the "door frame" which this seems to be. The entire assembly could be called a watertight door, but if one said "I walked through the watertight door" others might snicker. Still, in the testimony in the hearing about the [[Titanic] sinking, crewmen said they "went through watertight doors."[3] referring to the complete door and frame. Edison (talk) 20:46, 22 April 2008 (UTC)[reply]

Elephant relative

I'm trying to remember the name of a small furry animal which somewhat resembles a guinea pig, but, oddly enough, is most closey related to elaphants. If anyone knows its scientific and common name I would greatly appreciate it. BeefJeaunt (talk) 18:08, 22 April 2008 (UTC)[reply]

Never mind, found it: Hyrax. BeefJeaunt (talk) 18:15, 22 April 2008 (UTC)[reply]

Heat from Lights

Good day, I'm considering retrofitting the lighting system in my warehouse from Metal Halides to a T5 lighting sytem. I'm concerned that the reduced amount of heat generated by my lighting system(450 watts to 234 watts) will have an advverse effect on my heating bill during the winter months. Please tell me what effect such a retrofit would have on my heating bill, if any? Total number of fixtures = 200

thanks, Vinsanity3 —Preceding unsigned comment added by Vinsanity3 (talkcontribs) 18:54, 22 April 2008 (UTC)[reply]

Without doing any math or knowing your geographic location, heating fuel, local costs, etc. a good rule of thumb might be that you will have to pay as much for heating as you will save on electricity, for those months when you would need to have a 20KW electric heater running continuously to heat the building. Franamax (talk) 19:06, 22 April 2008 (UTC)[reply]
You absolutely cannot expect volunteers here to replace the need for licensed professionals in advising you on important engineering decisions such as this. That said, if the lights produce less heat, then all other things being held constant, that heat would have to be produced by the heating plant. If the spaceheating is by electricity, then there would be less of a difference than if the heating plant burned fossil fuel. Then consider the cost per unit of heat from the electricity versus the fuel used by the space heating system. If you needed a larger furnace, but the cost per unit of heat from the furnace was less than the cost of the heat from the present electric lights, someone needs to do some economic calculations and cost projections for you to make an intelligent choice. In a residence, with gas heat, one might actually consider putting the incandescent light bulbs back in in the winter to augment the heat from the furnace which is marginally undersized, but putting the compact fluorescents in in the summer to reduce the air conditioning load. Edison (talk) 20:22, 22 April 2008 (UTC)[reply]
It depends on what method of heating you're using.
  • If you're using electric heat, your bill will be about the same: electric lights and electric heaters are equally efficient at heating.
  • If you're using any other form of heating, your bill will probably be lower: in almost every part of the world, electric heat is the most expensive way of heating a building.
--Carnildo (talk) 21:40, 22 April 2008 (UTC)[reply]
The practical upshot is that incandescent bulbs put out very little ambient heat. Even then, let's just pick a random number: let's say running all the lightbulbs in an entire house increases the ambient temperature by 3 degrees during a day. That means in winter, you have a "free" 3 degrees of heat your furnace does not have to put out. However, in summer, that's an extra 3 degree penalty your cooling system has to compensate for! So, overall, it's not saving you anything. By switching to low-powered bulbs, you're objectively saving money on your electric bill, so I'd say the decision is simple there. -- Kesh (talk) 00:28, 23 April 2008 (UTC)[reply]
Sorry Kesh, disregarding the fact that the discussion is about halide bulbs, not incandescent, any electrical device puts out almost exactly what it says on the label - is it's a 40W bulb, you get 40 watts of heat. 8W of that is light, which is converted to heat when it meets its surroundings, 32W is indeed converted into sensible heat. Incandescent bulbs are in fact very efficient devices for producing ambient heat. Electrical devices are generally 100% efficient at producing heat, that's what they do (see thermodynamics). And by the numbers already supplied, we're talking about 20 kilowatts of heat, which is actually not bad, think 20 hairdryers.
You are somewhat correct in your argument about summer vs winter, but the OP is asking about a warehouse, which generally doesn't use any cooling in summer, usually you open the big doors at either end so the outside air can go through. The OP is trying to make an economic decision here - as Edison says, the question should be evaluated by a licensed professional who can best determine where the cost-savings are and how long they will take to be realized. We don't even know whether the OP lives in the desert or the Arctic, how well the building is insulated, we know nothing! Franamax (talk) 01:10, 23 April 2008 (UTC)[reply]
incandescent bulbs put out very little ambient heat Where do you get that idea kesh? Approximately 90% of the power consumed by an incandescent light bulb is emitted as heat, rather than as visible light. —Preceding unsigned comment added by 79.76.212.84 (talk) 00:54, 23 April 2008 (UTC)[reply]
And.... the visible light is absorbed, and becomes... heat. damn efficient heaters, light bulbs. i got a tiny nightlight bulb stuck to the wall beneath my thermostat, attached to a timer, which serves to crank the heat up and down for my daily routine, and you need only a tiny bulb not very close to the stat. and of course, everybody who raises chickens in a box in their kitchen knows you have to stick a small light bulb in there to keep them warm; if you used like 100 watts they'd end up kentucky fried before their time. odd but related note: the local Sears Hardware has normal looking light bulbs, labeled and sold as replacements specifically for easybake ovens. Gzuckier (talk) 19:17, 23 April 2008 (UTC)[reply]
Incandescent light bulbs produce an amazing amount of heat. Various numbers are bandied about for the efficiency/efficacy of incandescent lights, but perhaps 95% of the energy they consume produces heat rather than visible light. That is still a vast economic improvement over candles, kerosene lamps, or gas lights which were their predecessors. Right now compact fluorescent lights are the light bulb of choice, but in a very few years they should be displaced by LED lamps, which are thus far low in output and too expensive. Edison (talk) 05:47, 23 April 2008 (UTC)[reply]


Two points, Kesh: one is that summer and winter need not be of the same length or severity for the OP, and the other is that cooling is actually easier than heating here. See SEER: standard modern air conditioning systems need only one third to one fourth the amount of energy to run as they remove from the building. So, supposing (reasonably) that gas heat is used that costs half as much as electric heat, and neglecting the issues of heat distribution called out below (because fans are cheap), we have that in the winter the electricity to run the lights is half repaid by the reduced gas usage, and in the summer no more than a third again the power of the lights is required to evacuate their heat. Of course, none of this really matters; the only way that reducing the power of a lighting system could possibly hurt would be if the normal heating system were somehow less efficient (in money) than the electric heat provided by the lamps, in which case I would suggest replacing that heater with a box of light bulbs! Practically speaking, the retrofit would not be useful in a cold climate (where the heater was effectively always running) where the heater costs nearly as much as electric heat, because not enough energy will be saved to recoup the cost of the new lights. --Tardis (talk) 17:28, 23 April 2008 (UTC)[reply]
Reply to all: Yes, lightbulbs put out a lot of heat for their size. However, the amount of heat put off by a bulb is insignificant if you're looking at the overall household temperature. Now, I'll admit that I did overlook the fact the OP is talking about a warehouse. In that case, insulation is likely a bigger factor than the light bulbs when it comes to temperature, but large incandescents may have an effect in a building of that type. Still, I don't see it as enough to offset the long-term costs of changing to lower-powered bulbs. -- Kesh (talk) 20:47, 23 April 2008 (UTC)[reply]

I assume that your lights are mounted on/near the ceiling of your warehouse. Since warm air rises (see convection) the simple mathematical heat output of the lamps will not come close to the actual usable heat. Warehouses are not known for well insulated ceilings/roofs. My guess would be that a sizable portion of the heat from your light gets "lost". Your heating system on the other hand is designed to deliver heat where it's needed. --Lisa4edit (talk) 07:56, 23 April 2008 (UTC)[reply]

Very true, one of the first things I thought of. However, this is an industrial situation and that's where the pros come in to help with solutions - for instance, an 80,000 sq ft warehouse I'm familiar with was fitted with a ceiling-to-floor recirc duct (8-foot square) with a large squirrel-cage fan to effectively recirculate the air to floor level with two-year payback. An adjacent 180,000 sq ft mfg plant was fitted with a recirc duct and motorized ceiling vents for summer heat release. This was Southern Ontario (-20C to +30C) and not applicable to all climates. Point here is that we are really not in a position to give good answers about a building with 200 lights in it, but the OP might be able to get answers for free if they just ask the vendor the right questions. Franamax (talk) 10:42, 23 April 2008 (UTC)[reply]
Keep in mind that it will probably save money during the summer months, so if you install during spring then you will get the greatest cost benefit. As for heating, I imagine that waste heat from lights is probably a less energy efficient heating method than an actual heating system, so you will probably save some money during winter as well. Hope that helps! -- HiEv 10:41, 23 April 2008 (UTC)[reply]
of course, in winter the lights are on a lot more than they are in summer; when the AC is laboring away at noon, i haven't got the lights on; at 5 pm in the winter they go on, but in the summer not for a few hours more. on a related note, some folks point out that moving daylight savings time earlier as we do now may save electricity on lighting, but in the southern states it also means that folks come home when it's warmer and start up the AC an hour more every day. Gzuckier (talk) 19:17, 23 April 2008 (UTC)[reply]

Breathing question, again

When I hold my breath, and exhale a bit, I feel relieved, and as if I can hold my breath for longer. Have I somehow cheated? 81.93.102.185 (talk) 19:20, 22 April 2008 (UTC)[reply]

I don't know if you have cheated; what rules are you playing by? :-)
The human respiratory system detects the concentration of carbon dioxide in the lungs. Excess carbon dioxide results in a reflex to breathe. Note that this is contrary to what might be expected: your lungs can't measure the amount of oxygen in them; they measure the carbon dioxide. Getting rid of a bit of that CO2-laden air in your lungs tricks your central nervous system into thinking you don't need to breathe just yet. Weregerbil (talk) 19:49, 22 April 2008 (UTC)[reply]
Well, but you don't lower the concentration of CO2 in your lungs by breathing out; you do lower the absolute amount, but it's harder to explain how the breathing reflex would be triggered by the absolute amount. So there still seems to be something to be explained. My guess would be that something in the feedback mechanism says "oh, OK, he's complying, alarm bells can go silent now", and only a short time later notices that, no, the CO2 concentration is even higher than before. But it's just a guess. --Trovatore (talk) 20:04, 22 April 2008 (UTC)[reply]
Quite the interesting and logical take on it, Trovatore. That satisfies me, though I'm sure someone qualified out their bum will come by. :) 81.93.102.185 (talk) 20:06, 22 April 2008 (UTC)[reply]
I'll note here that it's also my understanding that breathing is controlled by CO2 concentration in the blood, not the lungs. Franamax (talk) 20:15, 22 April 2008 (UTC)[reply]
Good point. Another possible explanation that occurs to me is that perhaps when you exhale, you lower the pressure in your lungs (do you? not sure) and therefore the partial pressure of CO2. That could potentially lower the blood concentration of CO2. --Trovatore (talk) 20:19, 22 April 2008 (UTC)[reply]
It certainly feels like you lower the pressure. However, whether that lowers the pp(CO2) is not a given, wouldn't you need to look at the graphs of pp(CO2) vs. pp(N2) at constant temperature with varying pressure? Franamax (talk) 20:25, 22 April 2008 (UTC)[reply]
Exhaling is achieved through raising the pressure in your lungs. When you lower the pressure in your lungs, air flows in. Skittle (talk) 21:53, 22 April 2008 (UTC)[reply]
Well, but when you're holding your breath, you're blocking the airways so the air can't escape. When you exhale, you unblock them. As the air begins to flow, the pressure should diminish. --Trovatore (talk) 21:58, 22 April 2008 (UTC)[reply]
Qualitatively, when your lungs are most inflated, your body tissues are most distorted from their normal shape, so one might conclude that the pressure (return to normal shape) is at a maximum. Also if you fill your lungs and relax, you find that a lot of air exits quickly, which indicates a higher differential pressure. I'm thinking that the "feel-better" response to letting a little air out may just reflect a reduced sense of overall discomfort to the combination of increased need-to-breathe and overfilling-of-lungs. Whether there is an effect on actual blood CO2 concentration, I think depends on the various pp(X) graphs of gaseous and dissolved carbon dioxide and nitrogen. We need a chemical engineer/physiologist to weigh in here! Franamax (talk) 23:03, 22 April 2008 (UTC)[reply]
Nope, I'm sticking with the air in your lungs being at atmospheric pressure when air is neither entering or leaving. I can experiment with 'sealing' and 'unsealing' my throat, and when my lungs are full no air flows unless I force it out. If you have your lungs full 'to bursting', it may feel uncomfortable because you're stretching your body, but the air isn't at higher pressure than atmospheric until you start breathing out. Unless you 'seal' your throat and then deliberately try to breath out, but that wasn't what we were talking about. Breathing out relies on raising the pressure of the air in your lungs; if it didn't, air would not flow out. The 'tricking your brain for a moment' explanation seems more sensible. Skittle (talk) 12:06, 24 April 2008 (UTC)[reply]
I don't have a definitive answer either, but I will offer that this phenomenon has nothing to do with the partial pressure of CO2 in the lungs (so-called alveolar PCO2). Rather, ventilation has to do with arterial PCO2, and to a lesser extent arterial PO2 and pH.
My guess is that the phenomenon is related to the reflex that controls the depth of breathing. In human infants, this is regulated in large part by the Hering-Breuer reflex. Distension of the lung's air sacs activates stretch receptors that signal the brain to stop inspiring. I don't know of the equivalent reflex in adult humans, but I would assume it at least partly mediates the phenomenon you describe. --David Iberri (talk) 22:42, 24 April 2008 (UTC)[reply]

Right about global warming?

Is this right about global warming - the equations are wrong because of the boundary conditions, etc? Bubba73 (talk), 19:56, 22 April 2008 (UTC)[reply]

There's a few different answers here: the equations may indeed need correction, but that doesn't necessarily mean "global warming is wrong". Science is always in flux, there are many people looking at this subject, and if that scientist is correct, his ideas will soon enough be incorporated into the various models which simulate the warming process. It's turned out that it's not so simple as just changing the terms of an equation, the Earth system is much more complex, there is indeed controversy at the moment about warming at higher altitudes. However, keep in mind that 95% (at least) of the science does point to a warming trend. I'm somewhat disturbed at seeing this published on a blog, and there is a statement in there that '"runaway" greenhouse warming hasn't happened in the Earth's past', which I'm not sure is a true statement. Anyone else have a comment here? Franamax (talk) 20:38, 22 April 2008 (UTC)[reply]
Can't comment on the maths but Daily Tech isn't reknown for their analytic genious or fact checking. They come close to The Inquirer when it comes to tech news, and this isn't tech news. Also note the views of Stephen E. Schwartz who's work was mentioned in the blog Nil Einne (talk) 20:49, 22 April 2008 (UTC)[reply]
I at least understand what he is saying about differential equations and boundary conditions, but I haven't been able to find anyone that is an expert on the physics that agrees with him. Bubba73 (talk), 22:23, 22 April 2008 (UTC)[reply]
I did a quick search found a couple of comments indicating that a number of Miskolczi assumptions are flawed or baseless. Search for "Miskolczi" here or here (mainly comment #174) and that might help. There seems to be a lack of interest in debunking it since it only managed to get published in a minor (Hungarian) journal. I've also seen comments by others that Daily Tech is more interested in exciting sounding stories than accurate or reliable reporting. -- HiEv 10:27, 23 April 2008 (UTC)[reply]
Thank you for that information. Bubba73 (talk), 16:49, 23 April 2008 (UTC)[reply]
the math is pretty involved, alright, and i haven't seen it explained with the right and/or wrong boundary conditions filled in at any level I could follow, to prove or disprove it. on the other hand, from what i've seen in the past from "global warming debunkers", if i can't follow the math, you can rest assured that they sure can't, so these folks tossing the paper around as though they had some clue, are clearly just parroting what they've heard from folks just as ignorant. so, until I see it thusly explicated, i at least will have to suspend judgement on a mathematical proof or not; however, based on the sociological evidence of the "data" and logic presented by these guys in the past, specifically including Daily tech, it's not going to turn out to be a hill of beans. see also "the climate isn't really warming", "the sun is warming the climate", "it stopped warming in 1998", "mars is warming", "carbon dioxide isn't the most important greenhouse gas", "carbon dioxide is a requirement for life", "what about water vapor?", "what about china and india?", "you can't prove that carbon dioxide absorbs infrared outside the lab", "models can demonstrate anything you want them to", "Models can't demonstrate the current climate", "it's volcanoes", "it's cosmic rays", etc. etc. etc. etc. Gzuckier (talk) 19:31, 23 April 2008 (UTC)[reply]
I keep wondering about that Global Warming debate. Just because mathematicians could not solve Zeno's paradox (and some are still debating if that has been achieved) hasn't stopped people from moving from A to B and actually arriving. Just because we don't know all the answers to Global Warming doesn't mean it's not happening. --Lisa4edit (talk) 08:35, 23 April 2008 (UTC)[reply]
Actually, Zeno's paradox has been solved, and the solution is that an infinite sum can have a finite value (as e.g. the sum 1/2+1/4+1/8+1/16+1/32..., which converges to 1). Hence the time that the turtle can keep Achilles at bay also is finite, albeit decomposed into infinitely many time spans. --Stephan Schulz (talk) 09:00, 23 April 2008 (UTC)[reply]
SS, can you send me the computer printout that proves the solution by printing 1 at the end? Take your time, save paper and just send me the remainder when you give up :)
Lisa, you're right, uncertainties will never disappear (except that you will die sometime, but even then, how will you know for sure?) You have to act on probabilities in an uncertain world. We will never know for sure about global warming, we'd have to run the same planet through 20 or 30 different 100-year cycles to be sure. That's why the climatologists are so busy running simulations. because we only have one shot at the real thing. What we have to do is to make sure that our particular real-life experiment works out so that our kids and grand-kids can look back and say "you did good" :) Franamax (talk) 10:23, 23 April 2008 (UTC)[reply]
Ummm...you don't prove non-trivial things by computer addition, but by logical argument. In this case, the simplest argument is geometric. Take a line segment that is one unit long. Mark off a spot at 1/2. Then mark off one at 3/4, then one at 7/8, at 15/16, and so on. It is obvious that you can continue this process forever without ever reaching 1. The newly marked of segments are your new summands. The total marked length is your sum. There are many other proofs for the convergence of this series...I'll teach them at EUR 160/hour ;-). --Stephan Schulz (talk) 17:03, 23 April 2008 (UTC)[reply]
?? x=1/2+1/4+1/8+1/16+1/32... x+1=1+1/2+1/4+1/8+1/16+1/32..... 2x=1+1/2+1/4+1/8+1/16+1/32... therefore, x+1=2x. hopefully, you can take it from there. hats off to my high school math teacher. Gzuckier (talk) 18:24, 23 April 2008 (UTC)[reply]
Nice one. But you are destroying my market ;-) --Stephan Schulz (talk) 19:26, 23 April 2008 (UTC)[reply]
it's cause the euro is up and the dollar is down. Gzuckier (talk) 19:32, 23 April 2008 (UTC)[reply]
I blame this thread for getting me sucked into the black hole that is the Zeno's Paradox article and its talk page. When I started reading the pages and pages about trying to apply quantum mechanics to macroscopic objects my head just about exploded. You owe me a new head.Mad031683 (talk) 21:37, 23 April 2008 (UTC)[reply]
Bush will be out of office in January, I think. You can have his. The outside is somewhat used, but the inside is like new ;-). --Stephan Schulz (talk) 07:32, 24 April 2008 (UTC)[reply]

Will solar energy kill the Earth?

Sunlight:

  • Absorbed:
    • Absorbed by earth → becomes heat.
    • Absorbed by plants:
      • Used immediately by humans → becomes heat.
      • Stored as fixed caron.
    • Converted into human usable energy → becomes heat.
  • Reflected into space.

Let's say you are the Good King of Blah Blah Land, a cloudless tropical nation populated by honest-working and Earth-loving people. Your beautiful country is powered by a highly efficient and clean combined cycle natural gas power plant (60% efficiency). The aged power plant is very good but you really want to do the Earth a favor, so you replace it with a solar farm.

Your country has a desert covered with white quartz sand (50% reflectance, simplified figure). Your UFO-based national advisor provides you with several square kilometers of solar panels (10% reflected, 80% becomes heat, 10% becomes elctricity) shipped from Altair IV. Highly-paid union workers installed the panels in the desert in record time without any labor dispute and job injury.

Let's say your citizens need 1 unit of energy per day (they sleep all night and work all day). Your gas plant needs to consume 1.6 unit of energy to output 1 unit of energy. 0.6 unit of energy is released as waste heat. The unused desert absorbs 5 units of energy as heat.

Now to provide as much energy, your solar farm dissipates 8 units of waste heat ... At least you emit no CO2.

Will your good intention make the world an even hotter place? -- Toytoy (talk) 20:08, 22 April 2008 (UTC)[reply]

The total flux of solar radiation hitting the Earth and re-radiating into space absolutely dwarfs the amount of energy involved in electricity usage. The debate is not about heat-balance per se, it is about releasing long-stored carbon quickly into the atmosphere in the form of gases which enhance the greenhouse effect, i.e. as carbon dioxide. Using solar panels doesn't change the amount of heat input into the Earth system by the Sun, although it may change the wavelength of the energy re-radiated into space. Currently, it looks like reducing CO2 emissions is a good thing, so I'd say the king is doing good. Franamax (talk) 20:22, 22 April 2008 (UTC)[reply]
On second thought, burning fossile fuels only heats up the Earth one-time. The released CO2 makes the Earth more capable of absorbing solar energy ALL THE TIME. -- Toytoy (talk) 01:40, 23 April 2008 (UTC)[reply]
If the solar cells were on a giant dessert not a giant desert than it would get melted and people would not be happy.--Shniken1 (talk) 06:37, 23 April 2008 (UTC)[reply]
If you're only converting 10% into power and 80% ends up waste heat then you're doing it wrong. The heat should be used for power. Take a look at the Solar thermal energy for a design more suited to the situation you describe. At high enough temperatures you can get over 60% thermal efficiencies, which would match or beat your natural gas plant, but without the pollution or need for fossil fuels, plus it would reduce the desert's reflectance. So, it looks like solar energy saves the Earth in that situation. -- HiEv 09:41, 23 April 2008 (UTC)[reply]

Decent introductory treatment of quantum mechanics?

I've got a decent informed-and-interested-layman's understanding of quantum mechanics, coupled with a bit of ability and high school physics. Is there any reasonable (preferably free, online; lucid hard-copy textbooks are appreciated, though) treatment of the subject that would be both accessible to me but which would also provide sufficient of a formal introduction (maths doesn't overly scare me) that I could begin to make a little more sense of a lot of the information I'm finding. I've tried reading our articles on the subjects, but suffer the same problem I did trying to do the same with maths – not knowing where to start and a logical progression from there, I end up looking at one topic and having about a dozen more tabs open trying to understand other keywords in that topic. Cheers. Angus Lepper(T, C, D) 22:12, 22 April 2008 (UTC)[reply]

This is a continual problem with technical articles in Wikipedia. They are not accessible in that complex concepts are already assumed to be understood by the reader, or at best links are provided, but those links require following more links to understand, ad infinitum. I believe the cure is to have people who don't specialize in that field write, at very least, the intro for each article, so it won't remain knee-deep in techno-babble. If you feel up to it after learning the basics, please feel free to fix our articles on those topics. StuRat (talk) 22:31, 22 April 2008 (UTC)[reply]
The Feynman Lectures on Physics. Buy it, read it. Franamax (talk) 22:43, 22 April 2008 (UTC)[reply]
On that point, I agree with StuRat that technical articles tend to stay that way not because people don't try to make them clearer to readers, but maybe because the techs that write them don't trust accessible language so the problem persists. My experience, anyway. : / Julia Rossi (talk) 23:12, 23 April 2008 (UTC)[reply]
The problem is that accessible language is imprecise. When you get into complicated systems, it's important to keep the language precise so as to avoid confusion. That does, unfortunately, make it difficult for those not already knowledgeable in the field to pick up on the topic right away. -- Kesh (talk) 22:18, 24 April 2008 (UTC)[reply]
If the precise language is causing confusion, I suggest it be left for later sections where an in-depth discussion is warranted. A simplified intro should be always be included up front. For example, an "atom" can be described in the intro as "a clump of protons and neutrons in the nucleus, orbited by electrons in shells". In later sections the concepts of S, P, D and F orbitals and probability envelopes can be introduced. StuRat (talk) 01:26, 25 April 2008 (UTC)[reply]
You suggest it, I suggest it, many people suggest it, but it just doesn't stick because it hurts those used to precise language :) 79.66.99.37 (talk) 18:02, 25 April 2008 (UTC)[reply]
You could try to search for quantum mechanics lecture notes - they are sometimes online, like this one or this one. Icek (talk) 18:09, 25 April 2008 (UTC)[reply]
have you come across this article yet? SEems like a good place to get a broad udnerstanding on a variety of topics. Introduction to quantum mechanics --Shaggorama (talk) 08:13, 28 April 2008 (UTC)[reply]

Nitrogen

here is some stuff about nitrogen that i couldn't find and would like to know.

Atomic mass

  1. of protons
  2. of neutrons

Cost for sertan amount what family it belongs to —Preceding unsigned comment added by 76.14.124.175 (talk) 22:55, 22 April 2008 (UTC)[reply]

Atomic mass: 14.0067
Protons; 7
Neutrons; 7
See Nitrogen
Zrs 12 (talk) 23:03, 22 April 2008 (UTC)[reply]
Warning, Zrs has given some data but this may be the average, there are various stable and unstable isotopes which each have their own properties. Franamax (talk) 23:08, 22 April 2008 (UTC)[reply]
Oh yeah, my bad. The data I have given is for Nitrogen 14. There are other isotopes as well, however. See Nitrogen 15 for one example of another nitrogen isotope. Zrs 12 (talk) 23:11, 22 April 2008 (UTC)[reply]

How is carbon dioxide dissolved into water in pop?

When I was in chemistry today, my teacher said that carbonated water in pop is an excellent example of a gas dissolved in a liquid. However, I realized that since carbon dioxide is a nonpolar molecule, it can't possibl[y] dissolve in water, which is a superpolar molecule since it has hydrogen bonds. So then...how does carbon dioxide dissolve in water???--Dem393 (talk) 23:28, 22 April 2008 (UTC)[reply]

I'm sure there's more to the story than this, but my understanding is that the polarity means it doesn't dissolve particularly well, but it still dissolves. The pressure helps keep it in. Friday (talk) 23:32, 22 April 2008 (UTC)[reply]
A couple of things...firstly, solubility is not a matter of soluble vs. insoluble. It's actually a continuum of degrees of solubility. It is based on the chance at any given moment that two solute particles will stick to eachother when they pass one another, and the chance at any given moment that two solute particles stuck together will dissociate into the surrounding medium. When the first case is extremely more likely than the second, we call the substance insoluble, and when the second case is extremely more likely, we call the substance soluble. And so it actually completely untrue that something "can't possibl[y] dissolve in water." If it is unfavorable to dissolve, it will simply dissolve less. That said, carbon dioxide has a solubility in water of 1.45 g/L, over twenty times less soluble than table salt, but it does dissolve. And further, the polarity of the molecule is not all there is to it. The C=O bonds are themselves polar. So while there are no permanent dipole-dipole interactions taking place, the oxygens still have a limited ability to accept hydrogen bonds. We can see this even better in acetone; acetone is completely miscible with water, while its base hydrocarbon (propane) is quite insoluble (0.1g/L).
And to comment on the "pressure" comment above, yes, that does matter quite a bit. In the case of soda, the solute is basically a region of carbon dioxide gas above the uncarbonated soda. The solvent is the soda beneath. Increasing the pressure increases the chance that a molecule of CO2 will drop into the solution, increasing its solubility. And similarly, once you open the soda bottle and release that pressure, it will slowly decarbonate. Someguy1221 (talk) 23:45, 22 April 2008 (UTC)[reply]
Er, acetone might not have been the best example, as it's polar. But a similar argument holds for trans 1,2-dichloroethylene and ethylene (6.3g/L [4] vs. 35 mg/L). Someguy1221 (talk) 00:29, 23 April 2008 (UTC)[reply]
Thank you for that info. It really cleared up some confusion! I have another question: is the solubility of CO2 aided by its synthesis with water, as shown in this article?--Dem393 (talk) 00:15, 23 April 2008 (UTC)[reply]
It aids the solubility, yes, but most dissolved CO2 remains as is, most of the time (see Carbonic acid). Someguy1221 (talk) 00:29, 23 April 2008 (UTC)[reply]
CO2 itself is very insoluble in water, much less then 1.45g/L. But when that small amount does dissolve it is turned into carbonate and carbonic acid which are both much more soluble. Le Chatelier's principle tells use that this will cause more CO2 to be dissolved until an equillibrium between atmospheric CO2, dissolved CO2, dissolved CO32- and HCO3- is set up. Without these transformations CO2 would be much more insoluble in water than it is.
On a side note these reactions cause the water to become more acidic, this is seen in the worlds oceans as atmospheric CO2 concentrations increase. Also temperature greatly affects this equilibrium, as oceans get warmer less CO2 can be dissolved, so more ends up in the atmosphere, and the planet gets warmer and less CO2 can dissolve etc until we all die...--Shniken1 (talk) 00:27, 23 April 2008 (UTC)[reply]
I have another question, but first, could you please clarify what CO32 is? Ok, now here's my question: in chemistry class, I also learned the solubility guidelines. One of the guidelines say, "Most carbonates...are insoluble, except those of sodium, potassium, and ammonium." Carbonic acid is a carbonate, and the hydrogen ion isn't listed as an exception. Therefore, does that mean that carbonic acid isn't soluble in water? How would carbonation work then?--Dem393 (talk) 01:26, 23 April 2008 (UTC)[reply]
the 2- was an indication of a net charge. Carbonic acid is soluble in water as it dissociates - C02 + H20 ---> H2C03 ---> H+ + HC03- Wisdom89 (T / C) 01:29, 23 April 2008 (UTC)[reply]
An interesting historical note: See Joseph Priestley's notes from 1772 on carbonating water [5]. He is regarded as the inventor of soda pop. "If water be only in contact with fixed air (carbon dioxide) it will begin to imbibe it, but the mixture is greatly accelerated by agitation.." He noted that stale beer could be carbonated and revived. Edison (talk) 05:39, 23 April 2008 (UTC)[reply]
Hmmm...I was sure that Priestley got knighted..no mention of it in the article...--Shniken1 (talk) 12:01, 23 April 2008 (UTC)[reply]

Thank you all for your help.--Dem393 (talk) 00:59, 24 April 2008 (UTC)[reply]

Coming to a head

A quick question about abscesses as they occur in skin and superficial tissue. Often times when an abscess occurs it is at first several millimeters deep in the skin, and later on (whether from the natural progression or therapeutic intervention), the abscess "comes to a head", appearing, at least in part, closer to the surface. My question is this: what exactly is occurring during this process? Is the part which comes to the surface simply a fistulous tract, does the entire abscess itself move closer to the surface, or does the tissue between the abscess and the epidermis atrophy? Tuckerekcut (talk) 23:46, 22 April 2008 (UTC)[reply]

Seems rather like magma in a volcanoe working its way to the surface, with the volcanic eruption being akin to squeezing the zit or lancing the abcess. Edison (talk) 05:27, 23 April 2008 (UTC)[reply]
i always assumed that part of this was the continuous growth of new epidermis pushing out the old epidermis, which would result in the evolutionary advantage of making it that much harder for infections to penetrate. vs the serious infections that happen when they penetrate into the lower strata. Gzuckier (talk) 18:26, 23 April 2008 (UTC)[reply]


April 23

Sodium ethoxide (organic chemistry)

In my organic chemistry class I see this reagent used in various reactions. But I never knew what the sodium is for. Why can't the reagent just be OH minus? —Preceding unsigned comment added by 128.163.171.61 (talk) 01:15, 23 April 2008 (UTC)[reply]

Sodium is just a counter ion, would be the same if it were Potassium ethoxide. Ethoxide can be used as a base in organic (non-polar) solutions where Sodium Hydroxide (OH-) would not dissolve (as well).--Shniken1 (talk) 01:18, 23 April 2008 (UTC)[reply]
(ec)It's there for the same "reason" that one talks about the reactions of "just" OH- (hydroxide), but actually uses KOH or NaOH to get it--the K+ or Na+ is a counterion. It would be very hard to get a solution or solid that is "just" anions: each one repels all others. There were corresponding cations to balance their charge and give a net-neutral situation. DMacks (talk) 01:23, 23 April 2008 (UTC)[reply]
All the above said, ethoxide, one of the alkoxides, is often used because it is an extremely strong base, and some reactions require that level of basicity. In other reactions, such as some nucleophilic substitution reactions, the point of the reaction is to add ethoxide to another molecule through an ether-like bond. Someguy1221 (talk) 01:28, 23 April 2008 (UTC)[reply]
So counterions don't affect the actual reaction? Its only purpose is the preparation of the reagent (as mentioned by dmacks)? (original poster) —Preceding unsigned comment added by 199.76.153.227 (talk) 02:48, 23 April 2008 (UTC)[reply]
Right. You can never generate an anion without also generating a cation, and you can't remove the cation without replacing it with another one. Further, you usually actually want the counterion to do nothing, so you avoid complicating the reaction. Although you may find reactions in which both ions serve a purpose. Someguy1221 (talk) 06:47, 23 April 2008 (UTC)[reply]
Does it matter what counterion is to be used? For example, instead of NaOEt could one use KOEt? —Preceding unsigned comment added by 128.163.194.167 (talk) 15:36, 23 April 2008 (UTC)[reply]
I'd assume so not; except for a certain narrow set of biochemical interactions, and methods of seperating them, Na and K are chemically indistinguishable. Someguy1221 (talk) 17:16, 23 April 2008 (UTC)[reply]
I think Someguy1221 means that he assumes not. The counter ion should not influence the reaction much, it may only change the amount of dissociation that occurs in solution, but for Na and K the difference would be indistinguishable. This though probably won't influence the reaction because as the free EtO- is used up more of the NaEtO (or KEtO) will dissociate. So long as it is not involved in the reaction the counterion should not matter.--Shniken1 (talk) 02:15, 24 April 2008 (UTC)[reply]
Right, oops. Someguy1221 (talk) 03:34, 24 April 2008 (UTC)[reply]

(outdent) Except that the counterion very often is involved in the reaction when one looks closely. One uses bases to deprotonate other things, which usually renders those "other things" anionic, and hence they become associated with the counter-cation. It probably doesn't have much noticeable effect at the level of "here is how various functional groups react, this is the major product" and the types of reactions covered in most intro-orgo classes.

It's real though, because "the whole set of reactant molecules and ions" becomes "the whole set of product molecules and ions", not just "the basic piece deprotonating something" and the different metals have different properties. They might be better or worse at stabilizing an anion at a certain position in a molecule, for example, or have greater affinity for one or another structure. LiHMDS, NaHMDS, and KHMDS react "similarly", but when one _HMDS provides mediocre results, one often tries the others also instead of just scrapping the whole _HMDS type of bases.

Here's an interesting result: formation of the enolate of 2-heptanone can give two different enolates isomers (Δ1,2 vs Δ2,3). These two isomers obviously have different stabilities, and given the "more substituted double bond is usually more stable" as a first approximation, one expects to see mostly the Δ2,3 enolate (under thermodynamic conditions). Using Li+ as the counterion for the enolate (i.e., reacting the ketone with a base having a litium cation), one indeed gets 87%:13% Δ2,31,2. Switching to a K+ counterion, the thermodynamic ratio is only 58%:42% Δ2,31,2 (data taken from House, Modern Synthetic Methods) The more-substituted alkene is still "more stable", but not as much more stable. DMacks (talk) 03:07, 24 April 2008 (UTC)[reply]

I have another question: In various reactions, I see -OH + H2O used. Why is the counterion omitted there? Also, there are reactions that use H3O+, but there are others with H+,H20. What's really the difference between the two acid reactants? —Preceding unsigned comment added by 128.163.224.222 (talk) 19:42, 26 April 2008 (UTC)[reply]
A strong base "fully dissociates in water", so it looks like "OH- + something+" in solution (i.e., as used in a reaction). As above, when used as a base, the counterion ("something+") does not have a substantial effect in many reactions (it acts as a "spectator ion"), so it is often omitted on both sides of the reaction. Likewise, acids protonate water, so H+ in solution forms H3O+. But really, that thing reacts by transferring H+ onto something else...the water molecule is a temporary location or carrier for the H+, and the overall reaction is just H+ (coming from the original acid) protonating "something". It's all a matter of being as precise and technically correct as one wants without becoming overly complicated and obscuring the important part of the reaction with those technical details. DMacks (talk) 18:23, 27 April 2008 (UTC)[reply]

Water flow in pipelines

An overhead tank, connected to 4 floors of a building, each floor consisting 4 flats, has seperate water pipe line. How is it possible to control the water pressure when the pipeline has no safety valve to control the water flow? The pipeline has individual air outlet pipe only. Thanks, Narayan. —Preceding unsigned comment added by 59.178.113.204 (talk) 02:47, 23 April 2008 (UTC)[reply]

Unless there is some type of restriction in the pipe, such as a valve, I'd expect it to have a higher pressure at the lower floors. StuRat (talk) 02:53, 23 April 2008 (UTC)[reply]
Indeed that is the case. Most domestic appliances have a very wide range of pressures they can operate on, typically 0.5 bar to 10 bar. This allows operation with either a direct supply or from a header tank, usually without valves. In certain areas some houses have very high pressure feeds and need a pressure reducing valve, for example where there is a road going up a high elevation hill. In order to give adequate pressure to houses at the top of the hill, those at the bottom have a higher pressure than is usual. This was used by Yorkshire Water to excuse a high leakage rate, they said that if they could run the system on the same pressure as the London or Anglia regions the leakage rate would be lower than average, but the high pressure they need meand their leakage was above average -- Q Chris (talk) 07:26, 23 April 2008 (UTC)[reply]
In order to comply with British regulations, each flat should have its own valve where the water enters it. Instead, the property you describe would have to be wholly disconnected in order to change a washer. - Kittybrewster 09:07, 23 April 2008 (UTC)[reply]
This is a stop valve (stopcock), not a pressure regulator, i.e. it gives you the ability to turn off the water. In some cases non-return valves may also be fitted, but generally nothing affecting the pressure [6]. —Preceding unsigned comment added by Q Chris (talkcontribs) 07:47, 25 April 2008 (UTC)[reply]

Plumbing

what is meant by airlock system in waterpipe line? How does it work to control the water pressure? Thank you. Ranee. —Preceding unsigned comment added by 59.178.113.204 (talk) 02:54, 23 April 2008 (UTC)[reply]

An airlock (a plug of air) can be created to fill a gap in the volume of water. For example as a result of a minor leak. An airlock will seek a high place, eg the top of a radiator - which you then have to "bleed" with a radiator key. You may also need to fill the system up with more water to improve the pressure. - Kittybrewster 09:12, 23 April 2008 (UTC)[reply]
Do please register a name. - Kittybrewster 09:12, 23 April 2008 (UTC)[reply]
Or are you talking about an expansion vessel?? The expansion vessel is a metal container divided in two by a rubber diaphragm. One side is connected to the pipe work of the heating system and therefore contains water. The other, the dry side, contains air under pressure and you will find a car-tyre type valve for checking pressures and adding air. When the heating system is empty or at the low end of the normal range of working pressure the diaphragm will be pushed against the water inlet. As the water expands so the diaphragm moves compressing the air on its other side and giving rise to a moderated increase in pressure that you can see on the pressure gauge. - Kittybrewster 09:26, 23 April 2008 (UTC)[reply]
Outside of heating systems, water supply pipes often contain an air pocket (or stand pipe) to prevent water hammer, an overpressure problem. Rmhermen (talk) 12:28, 23 April 2008 (UTC)[reply]

Viruses/Bacteria

Hello,

I have a question:

Why do viruses and bacteria (except the "good" bacteria) cause their host to die if left un-treated by modern medicene. I thought the whole reason a virus spread was so that it could pass on its genetic material, and yet if the host dies it is unlikely that it will be able to pass itself onto another being. If viruses were like the worms Philip J. Fry got in Futurama, and instead enhanced the host body, surely that would be a much more effective way of passing the genetic material on?

84.13.26.33 (talk) 08:15, 23 April 2008 (UTC)[reply]

Actually only a small number of viruses and bacteria are likely to kill you if left untreated. The human immune system isn't that bad. For example, many millions of people catch the flu every year, a large proportion of them don't get any treatment. Very few of them die. Also, I'm sure someone will give a better answer then this but it might help if you think about the fact that most viruses are spreading well enough, even if they do eventually kill the host. Nil Einne (talk) 09:09, 23 April 2008 (UTC)[reply]
(Edit conflict) Only a small number of viruses and bacteria cause their host to die, it's not true to say that any untreated ones do (consider for example the hundreds of millions of people annually infected with the various strains of the common cold, almost none of which die from it). In essence, from a strictly 'selective' viewpoint the infection will cause whatever effect in the host that will maximise it's own survival, or the survival of its genetic material. In some cases this is the death of the host organism, as this may be the best way to rapidly multiply, or this is the way the virus or bacteria assists with its spread to other hosts. There's more to it than that, but other users can expand. --jjron (talk) 09:10, 23 April 2008 (UTC)[reply]
(Edit conflict) Actually, most (~99.99999%) viruses and bacteria don't kill the host. When is the last time you heard of someone dying of a cold? The immune system can usually take care of most damaging infections. Generally speaking, it's viruses and bacteria that are poorly adapted to the host that are normally the most dangerous, such as when they jump to a new species. Most viruses and bacteria, over time, usually evolve so that they do not kill the host. However, as with influenza, genes of two similar viruses may get mixed together in one host, producing a new, more dangerous strain. So, a version of flu adapted to only pigs may get mixed together with a version of flu that infects pigs and birds, producing a new version that infects pigs and birds, but is deadly to birds.
Furthermore, many multicellular species have developed symbiotic relationships with bacteria. Mitochondria have even become so symbiotic that most life is dependant on them. Some species even have symbiotic viruses.
Viruses and bacteria have also triggered many mutations in the ancestors of most species by adding some of their genes to other species, some of which were beneficial enough to spread throughout the population. So, the DNA of ancient viruses and bacteria live on in the species you see today in the form of beneficial mutations.
Still, it should be noted that in some cases the death of the host is irrelevant. If the disease manages to spread well enough before the host dies or continues to spread well after the host's death, then the host's death may not harm the bacteria or virus' spread enough to make evolving a less deadly strain occur. A long incubation period may have this effect, giving the host time to join another population, then killing the host during an infectious stage. This is rare though, since it is usually easier to evolve to adapt to the host. -- HiEv 09:17, 23 April 2008 (UTC)[reply]
I just have to point out a bit of an error.. The origin of mitochondria (and chloroplasts, in plants) is indeed thought to lie in symbiotic prokaryotes, but it's quite an overstatement to say that most life is dependent on them. Only eukaryotic cells (like our own) host mitochondria, and they represent only a fairly small branch on the tree of life. – ClockworkSoul 12:44, 23 April 2008 (UTC)[reply]
Actually, eukaryotes comprise a large branch of the tree of life, including animal, plant, fungi, and protist kingdoms and, with the exception of a few protists, all having mitochondria, while prokaryotes only include bacteria and archaea. I suppose we could discuss the estimated number of total species in each domain, their likely total individual numbers, or their theoretical overall mass on Earth; so let me amend my earlier statement to say "most (if not all) life you visibly encounter" is dependent on mitochondria. -- HiEv 07:09, 24 April 2008 (UTC)[reply]
It's important to remember that evolution is not directed (some opinions notwithstanding), so when you see suboptimal traits in nature it's not a matter of "what is the purpose of this feature?" so much as "this feature has not been bad enough to stop this species from surviving". It would be nice if humans could fly and run faster than deer and breathe underwater, but our suboptimal lack of those features has not been enough to do us in. --Sean 12:43, 23 April 2008 (UTC)[reply]

Consequences of helium depletion

The world's reserves of helium are being depleted, with US's reserves expected to end within a decade if we keep the extraction rate. What would be the impacts of such depletion on a worldwide scale? It makes me kind of sad to see all these stupid helium balloons around when this resource seems to be almost gone. — Kieff | Talk 10:33, 23 April 2008 (UTC)[reply]

Well, note that the economics of it should kick in when the amount left starts to actually be very low (that is, the price will rise so that it becomes worthwhile to conserve it). --Captain Ref Desk (talk) 14:42, 23 April 2008 (UTC)[reply]
If it ever becomes viable and implemented on a large scale Fusion power may provide more Helium than we know what to do with. Mad031683 (talk) 17:03, 23 April 2008 (UTC)[reply]
I seem to have grossly overestimeted the amount of helium produced in these reactors, since it seems only a few grams of fuel are used in these at a time. Mad031683 (talk) 17:30, 23 April 2008 (UTC)[reply]
It may help to consider the magnitude of what we're talking about. For example from helium:
Helium is commercially available in either liquid or gaseous form. As a liquid, it can be supplied in small containers called dewars which hold up to 1,000 liters of helium, or in large ISO containers which have nominal capacities as large as 11,000 gallons (41,637 liters). In gaseous form, small quantities of helium are supplied in high pressure cylinders holding up to 300 standard cubic feet, while large quantities of high pressure gas are supplied in tube trailers which have capacities of up to 180,000 standard cubic feet.
A new plant in Arzew, Algeria producing 600 million cubic feet (1.7×107 m³) came on stream, with enough production to cover all of Europe's demand
For example, the Saturn V booster used in the Apollo program needed about 13 million cubic feet (370,000 m³) of helium to launch.[2]
Also [7] "The Kennedy Space Center alone uses more than 75 million cubic feet annually." (not clear how much of this, is recovered).
According to [8] we currently produce about 25k tonnes of CO2 from power plants and we would produce about 10 million less of helium (it's not clear if we're talking about weight or volume but presume weight since that's what they gave). Even if this figure is inaccurate and it's only 10000 less and allowing for a 40 fold increase in our production from power plants, thats only 100 tonnes of helium per year (at room temperature and ATM). With the Kennedy Space Center using ~380 tonnes per year, that's only 1/4 of their annual usage Nil Einne (talk) 19:52, 28 April 2008 (UTC)[reply]


IIRC, the helium reserve was established when the military uses of blimps had a greater impact than is current, and we didn't want to be at the mercy of foreign governments. Gzuckier (talk) 18:28, 23 April 2008 (UTC)[reply]
I think the OP was refering to the sources of helium being depleted, not the stockpile of helium that was already extracted. Mad031683 (talk) 18:45, 23 April 2008 (UTC)[reply]
Most places that use a lot (or any)liquid helium have return lines to capture the boil off so it can be reused, this will become more important as it becomes more expensive. You may see some people turn to hydrogen instead.--Shniken1 (talk) 02:08, 24 April 2008 (UTC)[reply]

Reduced lung capacity and enlarged toncils symptoms

This question has been removed. Per the reference desk guidelines, the reference desk is not an appropriate place to request medical, legal or other professional advice, including any kind of medical diagnosis, prognosis, or treatment recommendations. For such advice, please see a qualified professional. If you don't believe this is such a request, please explain what you meant to ask, either here or on the Reference Desk's talk page.
This question has been removed. Per the reference desk guidelines, the reference desk is not an appropriate place to request medical, legal or other professional advice, including any kind of medical diagnosis or prognosis, or treatment recommendations. For such advice, please see a qualified professional. If you don't believe this is such a request, please explain what you meant to ask, either here or on the Reference Desk's talk page. --~~~~
As per the guidelines you ignored at the top of this page, we cannot and will not help you with medical, legal or other professional advice. Go see your doctor. -mattbuck (Talk) 14:44, 23 April 2008 (UTC)[reply]

ir absorbtion, polar vs nonpolar molecules, methane?

ok; i see that infrared absorbance requires a dipole in the molecule to interact with the electric field, so that N2 or O2 is transparent, but CO2 or H2O isn't. But, recalling that the CH bond is nonpolar, how does CH4 count as a dipole and therefore IR absorber (and therefore greenhouse gas as we all know, but that's not what I'm interested in, just where is the dipole in the CH bond)? Gzuckier (talk) 18:34, 23 April 2008 (UTC)[reply]

Perhaps I am misunderstanding something here, but there are no C-H bonds in CO2 or H20. However, the C-O bonds in CO2 are highly polar, and the O-H bonds in H20 are somewhat polar, and therefore CO2 and H20 interact with infrared wavelength radiation. --Bmk (talk) 20:56, 23 April 2008 (UTC)[reply]
C-H bonds are not "non-polar" either--the atoms do have different electronegativity--they're just much less polar than C=O, O-H, etc. Minor nit, Bmk: The O-H bonds in the "H2O" water molecule aren't usually considered "hydrogen bonds". That term is more appropriate for the interaction between the H on one water molecule and a different oxygen atom than the one to which it's normally bonded. DMacks (talk) 21:06, 23 April 2008 (UTC)[reply]
Yeah C-H bonds are polar, as are all bonds between different elements. You may be confused because Methane and other hydrocarbons are non-polar molecules, this is because the polarity of the bonds are balanced out due to symmetry. CO2 is a non-polar molecule as well.--Shniken1 (talk) 02:03, 24 April 2008 (UTC)[reply]
Good point DMacks - i've deleted the misleading link to reflect that. Thanks! --Bmk (talk) 03:04, 24 April 2008 (UTC)[reply]
thanks to all, answers my curiosity perfectly.Gzuckier (talk) 14:20, 24 April 2008 (UTC)[reply]

Cellphones: signal strength

I've read somewhere that the poorer signal strength a cellphone receives, the stronger signal it transmits. Now, I live in such a place that my phone shows "Network search" most of the time. (Except when I'm at school.) Does it mean it is dangerous to carry the phone around, when there's no signal? --grawity 19:24, 23 April 2008 (UTC)[reply]

Dangerous? No one really knows for sure yet, see Mobile phone radiation and health. But yes, cellphones will emit more transmitted power as necessary to reach the cell site; the cell site actually commands the changes in power as needed. In the old analog AMPS days, hand-held cell phones topped out at 600 mW of power (and "bag phones" and car phones topped out at 3W); I don't know if the 600 mW limit is still true in the digital age. (The safety article I mentioned says 2.0W peak for a GSM handset, but "peak power" probably isn't comparable to AMPS power (which was continuous)).
Atlant (talk) 20:45, 23 April 2008 (UTC)[reply]
The phone isn't going to keep ramping up the signal indefinitely, and they max out well before they get to anything that modern science considers dangerous. (Microwaves appear to be pretty harmless, except in really high doses.) Can you imagine a situation where a consumer phone might be allowed to transmit dangerous amounts of anything, but only to people not living in cities?
That's not to say that modern science mightn't be wrong. There's still a lot of research being done on the topic. While, it's pretty clear they don't cause cancer, but there are still all sorts of other horrible things that scientists are still studying in relation to phones. However, there's no way the Reference Desk is going to give you an answer that's more correct than current scientific consensus, so we really can't theorize much beyond that. APL (talk) 05:36, 24 April 2008 (UTC)[reply]
It's not necessarily more dangerous, but it will certainly drain your battery a lot faster! -- Kesh (talk) 22:33, 24 April 2008 (UTC)[reply]

April 24

any botanists here?

Our Rosette (botany) article sucks. Can someone (a) fix the wording (especially the confused second sentence), and (b) suggest some other common examples? (I'll be glad to try to take some more pictures. At the very least I'll have to try to take a better picture of a dandelion.) —Steve Summit (talk) 00:33, 24 April 2008 (UTC)[reply]

Hi Steve, after seeing your note I expanded the article. From this[9] it looks like succulents come into it in some way. Since they're lumped under "borderline" cases you might need to check it. What's wrong with the dandelion pic? Maybe a cross section would be nice. Julia Rossi (talk) 03:41, 24 April 2008 (UTC)[reply]
Thanks! But (not to sound like a schoolmarm grading your work, I hope) this sentence:
Often, perennial plants whose foliage dies and the remaining vegetation protects the plant.
is still not a sentence, and this sentence:
This is an example of a modified stem.
has what my schoolmarm always used to call a Vague Pronoun Reference, i.e., what does the 'This' refer to?
The picture is small and fuzzy, and you can't really tell what's going on; you can barely tell it's a dandelion, and it's not obvious what the "rosette" form is. I'll try to take a better one. (Lord knows, I've got plenty of subjects!) —Steve Summit (talk) 03:50, 24 April 2008 (UTC)[reply]
Yeah I was working on it, so you need to peek again and fix anything funny. Ha. I'm in a dandelion kingdom too. : ) Julia Rossi (talk) 03:58, 24 April 2008 (UTC)[reply]
Okay, I've got the wording mostly fixed up. One more question: what is it that's explained in the second paragraph under "Form"? —Steve Summit (talk) 04:20, 24 April 2008 (UTC)[reply]
Form : takes in the structure, relationship of the parts and variations within it throug examples. I've never collab'ed with anyone before this is fun. Julia Rossi (talk) 04:41, 24 April 2008 (UTC)[reply]
I went and monkeyed with it. I took out Paris quadrifolia because it has a whorl, not a rosette. There was some odd cut-and-paste detritus that I tried to sort out with me knowing approximately jack about plants. I can't believe a fern belongs here. If I remember my high-school biology, and I sincerely doubt it, ferns don't even have leaves. --Milkbreath (talk) 10:49, 24 April 2008 (UTC)[reply]
Hi Milkbreath, be gentle with the little article and put that bushknife down. As it happens, ferns do have leaves see[10] for the difference between the word "frond", "leaf" and the historical background. The ref for the fern also uses rosette for its structure. High school biology is very much subject to ideas in the section "Buffer for speech" above. In the article whorl it also applies as a less technical, alternative description of rosette. not – In some examples I got there was a diff between the rosette and leaves that grew higher on the stem. So when you say you know "jack" and monkeyed with it, is that helping? Julia Rossi (talk) 11:17, 24 April 2008 (UTC)[reply]
I'm exaggerating my ignorance out of humility, I hope. I'm a fast study, and I poked around a bit yesterday before giving up and praying for a real botanist. Are you indeed a botanist? I'm impressed. If so, I'll sheathe my machete until the facts are sorted out, at which time I'll timidly knock and humbly beg to be allowed to take a tiny snip here and there with my little pruners in your dreams. If not, stay a few steps behind me till I've cleared a path. --Milkbreath (talk) 11:49, 24 April 2008 (UTC)[reply]
In flinging my body across the fern bit so hope didn't squash it, or you. I knocked the article together after SirSummit's plea so it's hardly seamless as yet. Botanist? More like horticulture drop-out. Bush hats all around, your turn -- Julia Rossi (talk) 11:57, 24 April 2008 (UTC)[reply]
I've hacked a clearing into the article talk page where we can sit and sort this out over tea like the civilized people we all strive to be, especially the strivers. Thanks for the hat. --Milkbreath (talk) 12:13, 24 April 2008 (UTC)[reply]
I'm being humble too because early on, SirSummit called me a botany goddess and I forgot. Make mine Lady Grey black no sugar. Striving is all – collabaway, m'man.  ; ) Julia Rossi (talk) 12:45, 24 April 2008 (UTC)[reply]

Impacts, Ballistics, and Mass.

I am trying to ascertain the size and magnitude of damage a 'Orbital Weapons Platform', falling from orbit, would cause in terms of craters, etc. Given the object has roughly the mass of a big avg. city bus, possibly 50-100% denser, whatever is feasible, it's a hypothetical modern-era object that fell from orbit, I'm sure there is some natural curve for velocity here, it obviously couldn't be moving the speed of a comet or some-such. How high would the impact crater walls be? Is it feasible it has rutted up a trench in it's wake, on say a moderately shallow trajectory? Would any of it actually survive? It's a tough military-grade object. —Preceding unsigned comment added by 68.221.126.35 (talk) 00:38, 24 April 2008 (UTC)[reply]

Here's a start: Impact depth. As to how large the impacting object would actually be after travelling through the atmosphere: depends. Franamax (talk) 01:18, 24 April 2008 (UTC)[reply]

Not sure how to respond in this area. That helped some, I just don't have the math background to really comprehend the outcome. Some advice in that regard would help greatly. —Preceding unsigned comment added by IamSamoth (talkcontribs) 02:28, 24 April 2008 (UTC)[reply]

I'd ballpark it as approximately a Skylab-type impact. Pieces survive, but there's no crater to speak of. An orbital weapons platform is not a single monolithic hunk of metal, and is not designed for re-entry, so expecting it not to break up while falling is unreasonable. — Lomn 02:49, 24 April 2008 (UTC)[reply]

You might enjoy our Kinetic bombardment article (which includes the concept of "Rods from God").

Atlant (talk) 12:48, 24 April 2008 (UTC)[reply]

Hi. As for the impact, the Asteroid impact simulator from LPL Arizona may be helpful, but it is more accurate in depicting asteroid and comet collisions. Thanks. ~AH1(TCU) 00:57, 25 April 2008 (UTC)[reply]
Atmospheric_reentry#Uncontrolled_and_unprotected_reentries has a picture of MIR breaking up and burning up on its way down. Too bad they put it into the Pacific, but luckily for us, Skylab hit Australia. It says "Of satellites that reenter, approximately 10-40% of the mass of the object is likely to reach the surface of the Earth." If you want your space thingie to be more robust than the average satellite, you might bump that to, oh, 50 to 80 percent. Earth_orbit#Geocentric_orbit_types shows an orbital speed of 8 km/s for low orbit, so that's your initial speed. Reentry capsule shows an impact speed of "over 300 mph" for a Soyuz 1 capsule whose chutes failed to open. This is roughly the speed of a rather slow-moving airplane, so you can get some idea of the ground deformation you might expect when it hits. Impact crater deals with high-speed collisions, much, much faster than you get with falling man-made junk, so the crater made by junk would not be round like the crater made by space objects. A low angle combined with sloping ground could easily produce a furrow. --Milkbreath (talk) 01:39, 25 April 2008 (UTC)[reply]

Wow! Thanks all! Great info, awesome calulator! :) TYVM! —Preceding unsigned comment added by IamSamoth (talkcontribs) 13:25, 26 April 2008 (UTC)[reply]

JSTOR access

Any eggheads here with a JSTOR subscription? Help is needed here. Franamax (talk) 00:54, 24 April 2008 (UTC)[reply]

This has been done by Someguy1221, thanks Someguy, double-plus-good of you.
A general note here, are people aware of Wikipedia:WikiProject Resource Exchange where you can ask for subscription-only content from volunteers? I'd encourage anyone with such access to list there. Thanks! Franamax (talk) 04:04, 24 April 2008 (UTC)[reply]
It occurs to me that page could be much better advertised than it is, considering the relative dearth of posts there. Someguy1221 (talk) 04:22, 24 April 2008 (UTC)[reply]
Thanks - it's now in my notebook. Julia Rossi (talk) 04:58, 24 April 2008 (UTC)[reply]
I'm actually stunned by my discovery of that project. So many times, I've chased a lead, only to end up at a "subscription needed" roadblock. It should be way more well known. Any ideas on how to publicize it, to get volunteers and requesters, are welcome. I've started out at WikiProject Science. Franamax (talk) 05:35, 24 April 2008 (UTC)[reply]
Even slipping it into relevant moments for the compelled-to-lookup refdeskers, or at stumped talk pages would make a difference. It did to me, Julia Rossi (talk) 05:48, 24 April 2008 (UTC)[reply]

Evolution

Who coined the term "evolution" (I suspect it was either Lamarck or Darwin, but I'm not sure)? BeefJeaunt (talk) 01:32, 24 April 2008 (UTC)[reply]

Keep in mind that the definition of "evolution" applies to much more than the evolution of the species. According to Merriam-Webster, the term originated in english in 1622, from Latin. Another question might be, "Who was the first to use the term 'Evolution' in its biological context?" You may find History of evolutionary thought interesting. Someguy1221 (talk) 04:01, 24 April 2008 (UTC)[reply]
Though used in early Greek thought, this[11] cites Albrecht von Haller (1708-1777), "coiner of the word 'evolution' in biology", and refers to Wikipedia. (Though I wouldn't put it past von Haller, couldn't find it in the tl article.) Julia Rossi (talk) 04:23, 24 April 2008 (UTC)[reply]
Another peek shows his backup: "It was coined in 1744 as a biological term by the Swiss biologist Albrecht von Haller (1708-1777), to describe the preformationist theory that embryos grew from homunculi enclosed in the egg or sperm (Gould, 1977, pp.28-29, 1978, p.34). "Evolution" in this sense meant the progressive unfolding of structures that were already present in a prepackaged form (Bowler, 1989, p.9; Jaki, 1988, p.189)." In case you're working on the article Evolution (term) -- Julia Rossi (talk) 04:36, 24 April 2008 (UTC)[reply]
It's worth noting that Lamarck referred to his theory as transmutation, whereas Darwin only uses the word once—as a verb, at that—in the entirety of Origin of Species, and that's the very last word of the book ("There is grandeur in this view of life, with its several powers, having been originally breathed into a few forms or into one; and that, whilst this planet has gone cycling on according to the fixed law of gravity, from so simple a beginning endless forms most beautiful and most wonderful have been, and are being, evolved."). The term would have been, however, quite familiar to Darwin's public by the time he was writing Origin of Species, as it was used in precisely this way all throughout Vestiges of the Natural History of Creation, which was an immensely popular pre-Darwinian book describing something akin to biological evolution (taking its cue from stellar evolution—"We see a gradual evolution of high from low, of complicated from simple, of special from general, all in unvarying order, and therefore all natural, although all of divine ordination"). --Captain Ref Desk (talk) 13:58, 24 April 2008 (UTC)[reply]

Nitro

What is the cost of nitrogen in it's gas state? —Preceding unsigned comment added by 76.14.124.175 (talk) 04:45, 24 April 2008 (UTC)[reply]

Try this link [12]. Wisdom89 (T / C) 04:50, 24 April 2008 (UTC)[reply]
This might be useful to you too [13]. Wisdom89 (T / C) 04:51, 24 April 2008 (UTC)[reply]
(e.c X 2) $33 (AUD) for 'High Purity' in a size G from BOC plus cylinder rental, I think this is a bulk price though..--Shniken1 (talk) 04:54, 24 April 2008 (UTC)[reply]
Gaseous nitrogen is available very cheaply as long as you don't mind it being mixed with 20% oxygen and various other trace impurities. Gandalf61 (talk) 09:30, 24 April 2008 (UTC)[reply]
:-) StuRat (talk) 01:06, 25 April 2008 (UTC)[reply]

Gecko and Spiders (Melbourne, Australia)

What species am I? (flickr)
Black line = 10mm.

I'm wonder what species this gecko I found in my mailbox is.

Also there's a bunch of spiders I've taken photos of with poor or minimal IDs. Anyone want to have a go at giving an ID? This Spiders of Australia site is quite good, but I've given up. (link) —Pengo 05:04, 24 April 2008 (UTC)[reply]

This [14] little fellow seems to match the photo and the habitat. Richard Avery (talk) 17:47, 24 April 2008 (UTC)[reply]

Name of anticipatory effect in experiments?

Does anyone know what the proper term is for the effect where subjects try and anticipate the results the experimenter is trying to achieve, and respond accordingly? There's a specific term I'm thinking of, but I can't quite remember it. All suggestions welcome... thanks --131.111.135.84 (talk) 06:10, 24 April 2008 (UTC)[reply]

Subject-expectancy effect? — Kieff | Talk 06:24, 24 April 2008 (UTC)[reply]
That's it - and not sure how I missed it in the first place. Thanks! --131.111.135.84 (talk) 09:49, 24 April 2008 (UTC)[reply]
Sometimes known as demand characteristics and also see Hawthorne effect. --Mark PEA (talk) 15:50, 24 April 2008 (UTC)[reply]

Variegation in wild pigeon populations

I live on the 20th floor of an apartment complex in the centre of Sydney and feed various birds from my balcony, including lorikeets and pigeons (even though the feeding of pigeons is frowned upon). The lorikeets are very beautiful, feisty, and demanding, but are picky eaters. Nevertheless, I feed as many as I can to encourage them to live in the city. The pigeons will eat any old bread or seed, but they are numerous, and I only feed “special” ones which have particularly striking plumage, or are especially friendly. Every day, I see a large flock of these pigeons—many of them my own visitors—hanging about the park outside the complex, and being fed by residents and passers-by. I am no pigeon fancier, but I have noticed that our local flock seem to have a much wider variety of plumages on show than flocks in other areas. Hardly two birds look alike, and several have stunning red and white necks that made me think at first that they must be products of some interspecies mating. This is very different to other flocks I have seen in which the great bulk of their constituents could be clones.

Typically, in many other areas, people indiscriminately throw breadcrumbs at a seething mass of birds in a park, but in my local situation, residents can only feed a few birds at a time from their small balconies, and inevitably favour the prettier and more affectionate ones. I got the idea that perhaps the feeding patterns of the area had contributed to this variability. As this pattern of feeding has been going on for at least 40 years, I am wondering if there has been a form of selection pressure that has led to larger than normal variability in plumage patterns and colourings, and whether anything like this occurs in other pigeon groupings in comparable circumstances. Myles325a (talk) 07:47, 24 April 2008 (UTC)[reply]

Your observations are very interesting, although I could make no guess as to how valid they are. All I can say is there are many varieties of pigeons, as they have been domesticated for many centuries. The first chapter of Darwin's Origin of Species is largely dedicated to them, as examples of non-natural selection. Thanks for sharing. —Pengo 11:02, 24 April 2008 (UTC)[reply]
Yeah, it's hard to say. I would never underestimate the power of evolution to surprise; plus recently scientists have been observing more instances of short-term evolution. So, I'd say it's possible that feeding patterns have affected the plumage phenotypes of the local flock, but very difficult to say for sure. --Bmk (talk) 13:50, 24 April 2008 (UTC)[reply]
See Heike crab for another interesting example of unintentional artificial selection. StuRat (talk) 00:51, 25 April 2008 (UTC)[reply]

Aircraft noise

I live about 5 miles from a major airport. I've sometimes heard an echoing sort of aircraft noise that drops very rapidly in pitch from very high to very low in just three or four seconds, from silence into silence. I'm sure it's nothing to do with the Doppler effect as it happens so quickly and the sound is not overhead. I have come up with a theory based on the fact that (1) I live on a hillside (2) now that I come to think about it, I associate the sound with overcast weather conditions. Could it be that a sound wave is oscillating between the ground and the cloud base, and because this distance rapidly becomes greater as the aircraft moves "downhill", the frequency suddenly drops? Is the cloud base substantial enough to reflect this sound wave?--Shantavira|feed me 08:26, 24 April 2008 (UTC)[reply]

Weather conditions may make the sound more audible, but I think the rapid change in frequency is more likley to be due to changes in the aircrafts' thrust during their climb or descent phase, rather than any more complex explanation. Gandalf61 (talk) 08:41, 24 April 2008 (UTC)[reply]
I agree but gusting etc is also possible. You are proposing some sort of waveguide I think but I doubt the density difference moving from air to cloud is enough set one up. A cloud base (or temperature inversion) will reflect some sound but not enough to set up a standing oscillation I think. Also the frequency of audible sound is too high compared to the distances. --BozMo talk 09:09, 24 April 2008 (UTC)[reply]
Could you maybe be confusing the engines with the hydraulics? It could just be the hydraulics lowering the flaps.Luxosus (talk) 13:43, 24 April 2008 (UTC)[reply]
It seems quite unlikely that you'd hear hydraulics operating from that distance. — Lomn 15:07, 24 April 2008 (UTC)[reply]
Is the sound anything at all like an indecisive whistle (ie, with no clear association in terms of meaning connected to familiar whistle signals)? If you could somehow record this, I'm sure it would be quickly identified. Any further description of the sound would also help. 81.93.102.185 (talk) 17:14, 24 April 2008 (UTC)[reply]
You could be hearing the thrust reversers or spoilers activating on landing: they both change pitch and volume rapidly as the airplane slows down. --Carnildo (talk) 20:16, 24 April 2008 (UTC)[reply]

Annealing of Copper CuSn6

What is the procedure of annealing Copper CuSn6? Including at what temparature to heat how long and what temparature to cool and through what period of time? —Preceding unsigned comment added by 122.166.11.179 (talk) 11:01, 24 April 2008 (UTC)[reply]

Annealing (metallurgy) may be able to help you. Stifle (talk) 13:15, 24 April 2008 (UTC)[reply]

Speed of Tennis

The game of tennis can be played on different surfaces. The news often mentions that a grass surface is "faster" than a clay surface. What is meant by "faster" in this context? Astronaut (talk) 11:59, 24 April 2008 (UTC)[reply]

"Faster" here means that the ball loses less energy when contacting the ground, allowing it to move faster. Only applicable after a bounce, obviously. This similarly affects how quickly the players are able to move about the court. — Lomn 12:58, 24 April 2008 (UTC)[reply]
Grass also bounces much less, so the ball seems to come onto the player faster. Luxosus (talk) 13:46, 24 April 2008 (UTC)[reply]
As a tennis player, the term "faster" means how quickly the ball goes from side to the other, and how its trajectory is affected by the court (after the bounce). For example, clay courts are considered the "slowest" courts because the loose clay takes any spin from the ball and emphasises it; a topspin ball will bounce higher and further on clay than on a "hard-court" or grass. Grass, alternately, is the "fastest" because of the tendency of the ball to skid on the surface when it makes contact, making the trajectory more horizontally locked. Hard courts are the middle-ground, though even at the high-school level a easily discernable difference is commonplace depending on the roughness of the courts, the specific materials the court is made of, and the weather conditions. From what Lomn said, I shall digress. Each court has its own pace, but different players move better on different services. For example, current world #2 Raphael Nadal is the best mover on clay, while world #1 Roger Federer is the best mover on grass. It all depends on the style of movement the player utilizes. Though, I will give you the benefit of the doubt for that argument, since sliding is much easier to do on clay courts, and allows you to get to shots you otherwise wouldn't have gotten to.

Also, any court is subject to "speed" changes due to weather. Clay courts become faster when dry, grass courts slow down when dry. Hard courts are just a pain to play on when wet.

Hope I helped. —Preceding unsigned comment added by EWHS (talkcontribs) 16:26, 26 April 2008 (UTC)[reply]

polar molecular solids

the force of interaction in polar solid is that of a dipole-dipole interaction which is strong and thus they have a high melting point.yet they are in the form of gases or liquids in room temperature?why? —Preceding unsigned comment added by Arya237 (talkcontribs) 16:22, 24 April 2008 (UTC)[reply]

Everything is relative. The general patterns you mention only hold if all other properties are equivalent. And "room temperature" is an arbitrary cutoff you have chosen. Dipolar interactions are strong, but weaker than ionic interactions, and heavier molecules tend to have higher mp/bp. To see a specific trend, one must pick molecules that minimize other trends (i.e., compare tert-butanol and neopentane). The trend even holds for things that "are in the form of gases or liquids (at) room temperature", except that again it's the relative effect (the interaction makes the mp higher than it would be in the absence of that interaction): compare water and dimethyl ether and propane. DMacks (talk) 18:03, 24 April 2008 (UTC)[reply]

Insolubility of Sulphur in water.

Common salt and Sugar are soluble in water while Sulphur is not.Why Sulphur is insoluble in water?Explain. —Preceding unsigned comment added by Geniuskrishna (talkcontribs) 17:09, 24 April 2008 (UTC)[reply]

You may find the solubility and chemical polarity articles helpful. Note that sulfur is a pure element; salt is an ionic compound and sugar is can form hydrogen bonds; pure sulfur has neither of these features and so it does not readily dissolve in water. Nimur (talk) 17:28, 24 April 2008 (UTC)[reply]

Heart monitors -- what do they mean?

Can someone explain the symbols found on a heart monitor -- such as ECG, HR, PR, SpO2, Oxy CRG, ST, RESP, NIBP? --70.167.58.6 (talk) 17:30, 24 April 2008 (UTC)[reply]

That's the trouble with buying on e-bay there's never a users manual. Richard Avery (talk) 17:43, 24 April 2008 (UTC)[reply]

Here is what I know-

Hope this helps, Nimur (talk) 17:51, 24 April 2008 (UTC)[reply]

Thanks! Really helpful! What entry should all these abbreviations be in? --70.167.58.6 (talk) 20:46, 24 April 2008 (UTC)[reply]
Some of them should be in List of acronyms and initialisms, but they aren't. I looked for "ECG", for instance, and it wasn't there. I'll put it in now. --Milkbreath (talk) 21:19, 24 April 2008 (UTC)[reply]
You may be looking for this :) Regards, CycloneNimrodTalk? 22:23, 24 April 2008 (UTC)[reply]
Doh! (But some should be in both places.) --Milkbreath (talk) 23:14, 24 April 2008 (UTC)[reply]
Note also that ECG is sometimes written EKG, from the German language spelling. Nimur (talk) 05:35, 25 April 2008 (UTC)[reply]
I've only heard it called an EKG....--Shniken1 (talk) 07:01, 25 April 2008 (UTC)[reply]
Shniken1, I can guarantee you that, in Britain at least, it's called an ECG. They are the same thing. Regards, CycloneNimrodTalk? 07:50, 25 April 2008 (UTC)[reply]

Fish species with edible roe

Does anyone know of a good list of fish species with edible roe? Is most roe edible? At the moment I'm trying to figure out the edibility of white bass roe, with no luck so far. I've also failed to find similar information on other species. Thanks - Qatter (talk) 19:05, 24 April 2008 (UTC)[reply]

Cod, salmon, herring, alaska pollock and burbot are all roe-able, as it were; the article on roe has a list of different types, actually. As for white bass, they release their roe straight into running water, so i'm guessing its not eaten due to the difficulty of obtaining it. Ironholds (talk) 22:34, 24 April 2008 (UTC)[reply]
Most roe is collected by "milking" it from the fish directly, so I don't think that'd be an issue for commercial caviar or sushi consumption. -- Kesh (talk) 21:54, 25 April 2008 (UTC)[reply]

Is simply tasting the white bass roe something you'd be willing to try? If it looks at all appetizing, go for it. Vranak (talk) 22:43, 24 April 2008 (UTC)[reply]

Ion thruster in the atmosphere

The article about ion thruster points out that the mechanism are only practical in outer space. However, can/have we fly/flied with an ion thruster on Earth? (even if commercially is senseless). 217.168.3.246 (talk) 20:12, 24 April 2008 (UTC)[reply]

Nope. One of the primary reasons that ion thrusters are outer-space suitable is their very low thrust. You would as soon fly on Earth by flapping your arms. — Lomn 21:06, 24 April 2008 (UTC)[reply]
Okay, and its low thrust is due to technical problems that could be resolved or physically impossible? 217.168.3.246 (talk) 22:45, 24 April 2008 (UTC)[reply]
A thrusting force will accelerate an object in an atmosphere until the air resistance imposes an equally opposing force, then acceleration will stop. In the case of an ion thruster, the force is so small (atomic particles being ejected, very low mass output) that even the smallest breeze would be enough to counteract its thrust.
The low thrust isn't a "technical problem". Think of analogies with other technologies: A solid-fuel rocket motor can't be throttled. A turbine jet can't propel an airplane to hypersonic speeds. A propeller thruster can't propel an airplane to supersonic speeds. An ion thruster can't propel anything in an atmosphere. These aren't "technical problems", they are simply features of each technology. ~Amatulić (talk) 22:55, 24 April 2008 (UTC)[reply]
What if you ionize a lot of air with an air ioniser and then accelerate this mass of air? 217.168.3.246 (talk) 00:08, 25 April 2008 (UTC)[reply]
So, ion thrusters depend on the fuel being nearly wholly ionized (a large fraction of the propellant must be ionized in order for them to work). It is very difficult to fully ionize air at atmospheric densities. There is simply too much matter in air to ionize all at once. Even if you achieved full ionization of air, the resulting plasma would vaporize any nozzle or thruster chamber - no magnetic field we could ever produce in a rocket would be sufficient to contain or collimate an atmospheric density plasma. And an air ionizer merely ionizes a tiny fraction of the air - not nearly enough to produce thrust against the accompanying atmospheric drag; also the electromagnetic effects responsible for ion thruster operation would not work if the ions were in a dense mishmash of neutral atoms. Ion thrusters work by ionizing very diffuse streams of gas and accelerating the resulting plasma to high speeds. The short answer is, atmospheric ion thrusters aren't inherently impossible, but they are fully out of our forseeable grasp in terms of materials and technology. --Bmk (talk) 00:32, 25 April 2008 (UTC)[reply]
While people can't fly on them, there are "Ionocraft" which can fly using ion propulsion in the atmosphere thanks to the Biefeld-Brown effect and EHD thrusters, which are similar to but different than ion thrusters. You may have seen one on the Mythbusters anti-gravity episode. -- HiEv 05:23, 25 April 2008 (UTC)[reply]
That is. Is it theoretical possible to build a plane based on a EHD thrusters motor? 217.168.1.182 (talk) 10:41, 25 April 2008 (UTC)[reply]
Theoretically, maybe, but practically, I would say "no". The article shows a thruster 2 meters wide that produces only 2 Newtons of thrust (0.45 pounds of force). Critical specs for an airplane powerplant would be thrust/weight ratio, input power, and size.
Thrust/weight: terrible. The thrust produced is a small fraction of the weight of the device including power supply. To get sufficient thrust, your airplane is likely to be too heavy to lift off.
Input power: You need too much input power to get just a little bit of thrust. 100W per Newton is 1 horsepower to produce 7.5 lb of force; you're better off using that 1 horsepower to drive a propeller.
Size: The size of the power supply and thruster could well be bigger than the airplane you want to put it in.
If an EHD thruster can (as the article claims) achieve near 100% electrical to mechanical conversion efficiency, then maybe it's possible theoretically to build an airplane based on it, but acheiving that efficiency may mean even more sacrifices in thrust/weight, size, and input power. ~Amatulić (talk) 18:49, 25 April 2008 (UTC)[reply]

Musing over my Cider

Greetings, reference desk.

This afternoon, while nattering with a college friend over the finer details of next weeks presentation, I noticed something strange in my pint of cider. My friend and I decided to call it Bertie, and I'm hoping you can tell me what Bertie was... here are the specifics:

  • It was a pint of Bulmers.
  • It was full
  • Bertie appeared to be a tiny, pale yellow/white (though the cider may have stained him) speck, almost indiscernible, on the bottom of my glass (on the inside, in the cider)
  • He was stuck to the bottom
  • He emitted bubbles, about the same size as himself (very tiny), at a rate of about 8-10 per second, which all rose straight up to the top
  • He was the only thing emitting bubbles, the rest of the cider was still
  • He was stuck fast, I could rotate the pint 360 degrees and return him to his original position before the stream of bubbles had time to deviation the position at the other end from where the bubbles met the surface
  • I could smell nothing near the rim of the pint but cider
  • Towards the end of the pint, as I drank, it became shallow enough so that as I drank Bertie would be exposed to air, upon re-submergence, Bertie would not emit bubbles for a few seconds, then would gradually being and raise his bubble rate until he was at maximum bubblefication after 5-10 seconds or so.
  • Bubblefication went on at a steady rate (8-10 a second) throughout the 45 minutes that I was sipping this pint (slow day)

My friend and I theorized that either Bertie was some sort of organism that respired cider, or some speck of something or other that reacted with the cider to produce oxygen, CO2 or something like that.

So... who or what is Bertie?! A pint to whomever tells me!

Regards, SGGH speak! 22:42, 24 April 2008 (UTC)[reply]

Bertie is likely a little speck of crud that resulted from incomplete dishwashing. By providing a rough surface – in contrast to the smooth surface of the rest of the interior of the pint glass – Bertie acted as a nucleation site for the formation of bubbles. (A scratch or other defect in the glass would do the same thing.) The bubbles are just carbon dioxide slowly coming out of solution from the cider; no chemical reaction is occurring. TenOfAllTrades(talk) 22:56, 24 April 2008 (UTC)[reply]
Right. And as for the curious time delay after exposure to air and then re-submergence -- I'm guessing that what was happening for those 5-10 seconds was that the released CO2 was first filling Bertie's nooks and crannies, or forming a big enough bubble to overcome the cider's surface tension, at which point bubbles could again break free and form the stream. —Steve Summit (talk) 00:42, 25 April 2008 (UTC)[reply]
Another vote for a speck of dust acting as a nucleation site. This is far more likely than a living organism. Nimur (talk) 05:38, 25 April 2008 (UTC)[reply]
Interesting! Shame though, I thought perhaps I had discovered a new species (then murdered it when I finished my drink) oh well. Thanks guys! SGGH speak! 08:46, 25 April 2008 (UTC)[reply]

Cider is apple juice at a waypoint on the road to becoming Vinegar. In a program on semantics in the 1960's Mr. Wizard noted that one could sometimes find Mother in a bottle of vinegar, so perhaps Bertie is a very young Mother. See, for instance [15], where Mother/Bertie (formal name acetobacter) is said to be present in some cider vinegar. Was your cider a bit sour? [16] confirms the presence of acetobacter in cider. Edison (talk) 01:34, 26 April 2008 (UTC)[reply]

April 25

What's inside a snooker ball?

Are they solid plastic all the way through, or is there something else in the middle? --Kurt Shaped Box (talk) 00:42, 25 April 2008 (UTC)[reply]

See Billiard ball. DMacks (talk) 02:21, 25 April 2008 (UTC)[reply]
Billiard balls today are solid polymer, often a phenolic resin, polyester, or acrylic. This site has a cutaway picture. TenOfAllTrades(talk) 02:27, 25 April 2008 (UTC)[reply]
According to that site, the momentary "instantaneous" impulse, or collision, of stick against ball, has a "resulting friction temperature between ball and cloth can easily reach 250°c". What does this mean? I am not familiar with the term "friction temperature"; it seems like any heat would be absorbed by the relatively large heat-sink that is the mass of the billiard ball. Also, I was always taught that a billiard-ball is about the best example for a perfectly elastic collision, with virtually no energy lost to the impact deformation due to material stiffness. Comments? Is "friction temperature" just marketing-ese to sell more billiard balls? Nimur (talk) 05:42, 25 April 2008 (UTC)[reply]
When the ball skids across the felt the temperature on the surface of the ball would be 250C. The ball is a poor conductor of heat so any heat generated will not be transmitted will throughout the ball, thus the temperature at a local point can get quite hot. When the ball is skidding it is not elastic, when they are rolling and collide then it is close to an elastic collion.--Shniken1 (talk) 06:58, 25 April 2008 (UTC)[reply]
So the stripe goes all the way through? Interesting. I'd always thought that it was just a surface feature... --Kurt Shaped Box (talk) 22:54, 25 April 2008 (UTC)[reply]
Looks like it. Just like Blackpool rock —Preceding unsigned comment added by 79.76.217.249 (talk) 03:27, 26 April 2008 (UTC)[reply]
So, Snooker ball now redirects somewhere useful. Who was it that said that the Reference Desk did little to improve the encyclopedia? ;) --Kurt Shaped Box (talk) 19:50, 26 April 2008 (UTC)[reply]

Derivore

What is a derivore? 99.226.26.154 (talk) 00:43, 25 April 2008 (UTC)[reply]

Are you sure you don't mean detrivore? —Keenan Pepper 02:00, 25 April 2008 (UTC)[reply]
Or maybe an hors d'œuvre? Nimur (talk) 05:44, 25 April 2008 (UTC)[reply]

IDing Tanzanian termite-raiding ant

I know trying to ID insects from s.o.'s description can be an exercise in futility, but I hope I can provide enough details to narrow it down to the family at least.

I have come across colonies of similar, perhaps the same, large black ant in Tanzania, both near the coast on sandy riverbanks in forest reserves in Dar es Salaam, and upcountry in acacia scrub in the Rift Valley, within sight of Ngorongoro and the Serengeti. They're ~~1cm long, not particularly numerous (one colony a hundred or so, another maybe a thousand), and they emit a noise that sounds like rain when disturbed. Although I've never been stung, I've seen the effect on others, and it's clearly painful, though even a 4-yr old will stop crying after 5-10 minutes, and there's no swelling (from a single sting, anyway). However, they have one of the more painful ant stings in the area. They're diurnal hunters, and hit harvester termites on their forays. (Prob'ly other things, but termites is what I've seen.) In Swahili they're known as sungu-sungu, though there's so much regional variation in species and names that that might not be much good for an ID. They're larger than the siafu.

Anyone have any ideas?

Thanks, kwami (talk) 01:29, 25 April 2008 (UTC)[reply]

Hi Kwami, Sungusungu gets this: (Tanzania) they are (Kiswahili) "biting ants" and "In Swahili, Sungusungu denotes a highly cooperative and aggressive type of ant; in the Sukuma language, it means poison." in an article on vigilante groups[17]. Hard to find anything on them in entomology, except vaguely that they are meat-eating and get this, "small ant"[18]. Only the Carpenter ant is termite specific and widespread but nothing about origins. (addn) PS have you seen this[19]?Fwiw, Julia Rossi (talk) 09:18, 26 April 2008 (UTC)[reply]
Thanks. Carpenter ants are formicins, which makes sense since they may nest in the walls of houses, but it doesn't fit their sting — but then, I've never been stung, so maybe I'm wrong on that. They are the right size.
When you say 'termite specific', do you mean in nesting, or raiding?
kwami (talk) 10:13, 26 April 2008 (UTC)[reply]
Had in mind raiding. Julia Rossi (talk) 12:22, 26 April 2008 (UTC)[reply]

More Quantum Theory

According to Zeno's paradox#Does Quantum Theory solve the paradox?, it states that Planck length and Planck time are the smallest mesurable units of their respective dimensions. It then implies that space may be discontinuous and not infintely divisible, having the Planck length as the smallest unit of traversable space. How could this be? Where could I find more information about this? Furthermore, why is it (Planck length) the smallest mesurable unit of space, the Planck time the smallest mesurable amount of time? Why can nothing smaller be meaured? Thanks, Zrs 12 (talk) 02:50, 25 April 2008 (UTC)[reply]

Planck units are expressed in terms of relations between fundamental constants; our theories which use such constants don't make any sense if you start talking about units smaller (or in some cases larger) than Planck units. When you're thinking about things that small you can't let yours intuitive concepts interfere—length, time, matter, etc. don't operate on those scales the same way they do on more macroscopic, human scales. The easiest way to make sense of Planck units is to start by understanding the importance and implications of a quantum of energy (quantum), and then think about what it would mean to establish that sort of thing for other measurements. It's pretty unintuitive when you first come across it but there are pretty unimpeachable experiments that support it. --68.125.224.4 (talk) 03:54, 25 April 2008 (UTC)[reply]
Why can nothing smaller be meaured?. What are you going to measure it with? If the smallest measuring rod that can be constructed is a planck length and the smallest clock tick is the planck time, then it is not possible to construct a measuring instrument that can measure anything smaller. SpinningSpark 09:22, 25 April 2008 (UTC)[reply]
It does not automatically follow that you can't measure something smaller than the smallest measuring stick you can construct. It's easy to measure beyond the resolution of your measuring device. You can introduce some random noise known to average to zero and then measure the signal plus the noise and average over a large sample. The great part is that the noise can be several times larger than the signal, as long as it averages to zero you still get a measurement at greater precision to your measuring stick.
I'm not qualified to comment on this in regards to the Plank length, though. I imagine there are other difficulties that go beyond simple resolution restrictions on a measuring stick. APL (talk) 13:25, 25 April 2008 (UTC)[reply]
You assume that there exists something smaller to measure. If no such thing exists, it is impossible. For instance, if the charge on an electron were the smallest possible charge (I know about quarks, but am going to ignore them for this discussion) then your charge meter can only go up and down by whole electrons at the very best resolution that can be constructed. This is not some deficiency in my skill at constructing the meter. It is because the meter itself must necessarily use electron charge in the parts from which it is constructed. If you then add noise, you can only add noise in +/- whole numbers of electrons. You might get an average that is not a whole number but that does not mean that there is a fractional number of electrons in the box. It just represents the statistical error due to a less than infinite sample. SpinningSpark 10:02, 26 April 2008 (UTC)[reply]
You might want to read Heisenberg uncertainty principle, Planck length#Physical significance, and Planck time#Physica significance. The last section brings up some possible evidence to challenge this theory, but information is sketchy; you might want to follow up with the references. --Prestidigitator (talk) 18:34, 25 April 2008 (UTC)[reply]

Type of spider in Spider fighting

Can anyone identify the spiders in the article? These spiders are orb-weavers, roughly .5 to 1 cm long. They live in trees, power lines and tall grasses. They are found here in the Philippines. I posted this question on WP:SPI on August 2007 but didn't get any replies.--Lenticel (talk) 05:58, 25 April 2008 (UTC)[reply]

Hi Lenticel, I get a type of Argiope and there's one here[20] close-up. Addn: Aka a type of garden orb weaver[21]. Julia Rossi (talk) 01:37, 26 April 2008 (UTC)[reply]
The spiders looks more like Araneus (dull colored) than Agriope (larger and strikingly colored). I read List of Araneus species and found out that there are many species of Araneus in the Philippines. Well at least we narrowed it down to the genus level. Thanks Julia!--Lenticel (talk) 07:11, 26 April 2008 (UTC)[reply]
Ah, found it confusing colorwise especially sinceboth g's have their less vibrant types -- the Araneus I found looked highly coloured and ball shaped. Do they also like to web around power lines? happy searching,  : ) Julia Rossi (talk) 07:46, 26 April 2008 (UTC)[reply]
PS You've got a few Argiopes in the Philippines too – those spiders get around! Julia Rossi (talk) 07:51, 26 April 2008 (UTC)[reply]
Yes they do spin webs in power lines. Kids prefer these spiders as fighters as they are "brave" enough to show themselves openly to predators. Yes we do have Argiopes here which we call gagambang ekis (X spider) due to their X like decorations--Lenticel (talk) 11:13, 26 April 2008 (UTC)[reply]
A tip from an arachnidologist: that the size of a web shows how powerful a spider is – poor spider, Julia Rossi (talk) 13:06, 26 April 2008 (UTC) PS Thanks for the link to decorations - didn't know we had that article. JR[reply]

Shells

Hi, What is actually the shape of each shell of an atom ? Is it a 3-D spherical shape, or an ellipse like that of the planets, or something else? Thanks. 116.68.70.51 (talk) 10:37, 25 April 2008 (UTC) A 15 year old.[reply]

See Atomic orbital. It differs between difference subshell types. s-orbitals are spherical, but the others aren't as simple. 83.253.253.166 (talk) 10:59, 25 April 2008 (UTC)[reply]
The shapes are usually modeled as Legendre polynomials in spherical coordinates, because these forms are solutions to the governing equation of an electron. The easiest solution is of course the sphere, or s orbital. However, the higher order shells are very complex and may exhibit weird geometries (e.g. quantum effects) - for example, the electron's distribution function may be finite on both sides of a node and zero in between. Nimur (talk) 14:32, 25 April 2008 (UTC)[reply]

Thank you! 116.68.71.144 (talk) 13:12, 26 April 2008 (UTC)A 15 year old[reply]

Eyes

Hi! This is the same 15 year old again.........What I want to ask is, when an eyelash falls in the eye, somewhere in the white portion, I might not even notice it, but if it falls on the pupil, I would get a prickly stinging sensation immediately, even if it touches only a small portion of my iris and doesn't interfere with vision. Why is it so? I thought that the conjunctiva covered the front of the eye evenly. How should I know precisely when something touches only a certain portion?? (The conjunctiva doesn't have nerves, does it??) 116.68.70.51 (talk) 10:45, 25 April 2008 (UTC) A 15 year old[reply]

My understanding as someone who hasn't studied the eye, but has worn contacts for a very long time, is that the eye itself has no sensory nerves and doesn't notice anything on it. Your eyelid, on the other hand, is very sensitive and notices immediately if there is anything disturbing the smoothness of the eye. An eyelash "in your eye" is only noticed as it gets "run over" by the lid. -SandyJax (talk) 15:03, 25 April 2008 (UTC)[reply]
However, if you see the eyelash fall, that may trigger the "I've got something in my eye" feeling, too. So, lashes which fall in your field of view are more likley to cause a reaction. StuRat (talk) 15:27, 25 April 2008 (UTC)[reply]
There are no nerves (motor or sensory) in the sclera (white of the eye) so nothing can be felt on it. The pupil doesn't have any nerves either, but (as StuRat mentioned) I believe when the eye visually notices something on the pupil, it triggers a response; namely, pain to help you know it's there and the eye starts to secrete tears. Regards, CycloneNimrodTalk? 16:43, 25 April 2008 (UTC)[reply]
I have difficulty believing that there are no sensory nerves on the sclera or pupilcornea. If I hold my eyes open to let them dry out, they begin to sting, and that sensation is definitely on the surface of my eyes. If I take a tissue paper and poke it into different places on my eye, it tickles or even hurts. This also happens in the dark if I can't see what's happening. I'm skeptical that a visual cue will trigger a physical pain response localized on the eye surface. ~Amatulić (talk) 18:53, 25 April 2008 (UTC)[reply]

The pupil isn't part of the surface of the eye. Think of the surface of the eyeball as consisting of two parts: the white part, the sclera, and the clear part (which overlies the iris, lens, and pupil), called the cornea. The sclera has very little in the way of neuroreceptors, but the cornea is one of the most sensitive and densely innervated tissues in the body. Corneal abrasions are exquisitely painful. - Nunh-huh 20:07, 25 April 2008 (UTC)[reply]

The entire surface of the eye has sensory nerves that are used to trigger tear formation. I've done work on two studies involving sensory nerves on the eye. One found reduced sensation and reduced tears in patients who had LASIK. Another found that lack of sensory nerves on the surface of the eye correlated with early onset of glaucoma. -- kainaw 00:42, 26 April 2008 (UTC)[reply]

Cellphone signal

Someone told me today that numbers like 911, 112 and 999 (emergency numbers) can be called without any signal. I doubt very much that this is possible, but if it is, someone please explain how it can be? Thanks! Regards, CycloneNimrodTalk? 16:54, 25 April 2008 (UTC)[reply]

Perhaps what was meant was that emergency numbers can be called without a service plan; since a cell phone is, at a very basic level, a radio transceiver, a signal is necessary to make it work. --LarryMac | Talk 17:07, 25 April 2008 (UTC)[reply]
Agreed, they had the terms mixed up. Or you do. Either way, cell phones can call emergency numbers without any service plan. In fact, many communities have charities for abused women that give away old cell phones for free. They give them to women who may be in danger from their ex-husbands and so forth, so that if the need arises they can call 911 without having to pay for a service plan. see this, this, and this. Dismas|(talk) 17:17, 25 April 2008 (UTC)[reply]
From a user point of view it can seem as if they operate without any signal, and if you define 'signal' as the thing that your phone tells you it has (as is the way many lay people use it), then it does. Often when I don't have any 'signal' (I have no bars lit up) my phone says 'emergency only', meaning it can only be used to call 999 or 112. This is because there is no signal for my network, but other transmitters for other networks can be reached if necessary. 79.66.99.37 (talk) 17:33, 25 April 2008 (UTC)[reply]
QI tells us that on Christmas Day, lots of Americans call 911 to test their new phones because they have no network. -mattbuck (Talk) 14:53, 26 April 2008 (UTC)[reply]

I've been cleaning up the NAV disamb page and I just want to check whether the following is correct:

In anatomy, NAV stands for nerve-artery-vein, when all these follow a common pathway.

Zain Ebrahim (talk) 17:05, 25 April 2008 (UTC)[reply]

True, a useful mnemonic for the order of vessels and nerves on the front of the upper thigh, for example. A fuller version is NAVEL - (lateral to medial) Femoral nerve, femoral artery, femoral vein, empty space with lymphatics. I tidied up the NAV (anatomy) page a bit. -- Flyguy649 talk 01:47, 26 April 2008 (UTC)[reply]
And yes in shortform, writing "Femoral n.a.v" would mean all three. -- Flyguy649 talk 14:38, 26 April 2008 (UTC)[reply]

What causes dark circles under the eyes?

What causes dark undereye circles? --70.167.58.6 (talk) 18:54, 25 April 2008 (UTC)[reply]

The eye circle article has an answer, although it isn't cited with reference. ~Amatulić (talk) 20:30, 25 April 2008 (UTC)[reply]
The two explanations in the article for dark circles resulting from lack of sleep or fatigue seem utter bollucks. How would lack of sleep "make the skin pale" to the extent that the underlying blood vessels become more visible, but nowhee elso on the body? What layer of the skin would "become pale?" I thought pallor is more often associated with a decrease in peripheral blood flow. The other explanation is that "when one lies down since gravity can cause fluid to collect in the lower eye lid" again unsatisfactory because when I have stayed up all night, by definition I have not been lying down, and this assuredly results in dark circles. Edison (talk) 01:27, 26 April 2008 (UTC)[reply]

cockateils

moved from Village Pump -- Kesh (talk) 21:44, 25 April 2008 (UTC)[reply]

We have questions about weird things our bird does. what size are their hearts? our bird will look straight ahead at us and we can barly see his eyes...can he see us? also...he does this weird thing with his mouth. It looks like he's yawning, but he'll do this over and over again. & whats the best way to start petting him? He's used to us, but he doesnt like us touching him. Whats he doing? Thanks email removed —Preceding unsigned comment added by 209.33.106.106 (talk) 17:09, 25 April 2008 (UTC)[reply]

Guess you've seen our article Cockatiel - it's a start, especially about getting to know your little bird. Julia Rossi (talk) 12:26, 26 April 2008 (UTC)[reply]
Maybe by putting User:Kurt Shaped Box's name here, he will notice this question. Kurt is the resident bird/gull/budgie expert and may have useful insights. Nimur (talk) 14:36, 26 April 2008 (UTC)[reply]
This site [22] has lots of good information on bird behavior and training.--Eriastrum (talk) 16:12, 26 April 2008 (UTC)[reply]

April 26

Famciclovir or Acyclovir

Are there any instances in the literature of where short doses of fam/a/ciclcovir induced herpes zoster without prior infection? Also, adverse events such as peripheral neuropathy? Thank you.

75.73.61.30 (talk) 01:55, 26 April 2008 (UTC)Jonathan[reply]

Herpes Zoster is caused exclusively by the varicella zoster virus. Infection with VZV is necessary for herpes zoster, and thus neither aciclovir nor its derivatives (or anything, really) can induce this syndrome in an uninfected person. Peripheral neuropathy is not listed, per se as an AE in my dated (2004) PDR for acyclovir, though parasthesia is. Tuckerekcut (talk) 05:04, 26 April 2008 (UTC)[reply]
Thank you for answering. Kind of worded it wrong, inducing shingles in previously unshingled participants. I am aware dormant VZV is required. —Preceding unsigned comment added by 71.195.4.244 (talk) 20:12, 26 April 2008 (UTC)[reply]

Space and time as emergent phenomena

Kindly point out any references (books, webpages, etc.) on space and time as emergent phenomena. --Masatran (talk) 07:45, 26 April 2008 (UTC)[reply]

this sounds suspiciously like homework...hell, it's not even a question, jsut a demand for others to do research. GO TO A LIBRARY. If you can get to a college campus, try running a database keyword search, like on JSTOR or something. --Shaggorama (talk) 17:26, 26 April 2008 (UTC)[reply]
Please don't bite the questioners. This question is exactly what a reference desk is here for. --Sean 20:13, 26 April 2008 (UTC)[reply]

Science Without Numbers by Hartry H. Field or something similar?

I'm looking for Hartry H. Field's Science Without Numbers: A Defence of Nominalism in which, apparently, he describes physics without the use of any mathematics. It looks like it is out of print, and the used copies are rare and expensive. Since I'm in Brazil, a scientific and philosophical desert, this sort of thing is not really available in any place I've checked.

Is there any other way I could possibly get my hands on this book, or at least something similar? I'm very curious about this sort of approach. Thanks! — Kieff | Talk 08:58, 26 April 2008 (UTC)[reply]

Sorry, I don't know the book, but I always found Michael Faraday very inspiring. He achieved a high degree of scientific proficiency without much math. (Not that I hate math, just shows that it can be done.) Lisa4edit--71.236.23.111 (talk) 10:12, 26 April 2008 (UTC)[reply]

WH Smith in the UK claim to have it available [23] for £24.95 (that's about $50). Cheapest I've seen, the second hand copies seem to be selling for up to $200. I rather suspect though, that if you actually try and purchase the book from Smith's, it might turn out they don't actually have any. Other suggestions: you could try listing it on eBay as an item wanted - never know your luck; you could also ask here WP:Library. SpinningSpark 12:18, 26 April 2008 (UTC)[reply]
I don't think Faraday is the same thing by the way. Faraday was not actually against mathematics. Field apparently wants to expunge it from science. SpinningSpark 12:18, 26 April 2008 (UTC)[reply]
He's not trying to expunge mathematics as such, but rather mathematical objects (numbers, functions and such). In that book he shows how one can develop physics (he uses Newtonian gravitation as an example) with essentially its usual mathematical structure, but with all talk of dodgy non-existant objects replaced by talk of points in physical space and suchlike. A caveat to the OP: a JSTOR review has 'It should be added that this is not a popular work for the general reader; it is a technical work presupposing familiarity with current efforts to axiomatize physical theories by means of mathematical logic'. Algebraist 12:57, 26 April 2008 (UTC)[reply]
This may not help, but it may be worth a try:
Hartry Field, Silver Professor of Philosophy; Professor of Philosophy
Department of Philosophy / 5 Washington Place / New York, NY 10003
Phone: (212) 998-3795 / Fax: (212) 995-4179 / Email: hf18@nyu.edu
He is the author of Science Without Numbers (Blackwell 1980), which won the Lakatos Prize, of Realism, Mathematics and Modality (Blackwell 1989), and of Truth and the Absence of Fact (Oxford 2001). Current interests include objectivity and indeterminacy, a priori knowledge, causation, and the semantic and set-theoretic paradoxes.
Amazon has an offer for $145 plus a review which you may find interesting.
--Cookatoo.ergo.ZooM (talk) 12:49, 26 April 2008 (UTC)[reply]
Where in Brazil are you? There's an excellent book store in a lot of the larger cities called 'Livraria Cultura' that has a surprisingly good philosophy section, books in MANY languages, and will also order books for you. [www.mercadolivre.com Mercado Livre], Brazil's response to eBay might giv eyou some options too. Does amazon not deliver to Brazil? --Shaggorama (talk) 17:38, 26 April 2008 (UTC)[reply]
According to Worldcat, the university of Sao Paulo has a copy. -Arch dude (talk) 18:38, 26 April 2008 (UTC)[reply]
Yeah. Guess that's the best chance I have. I don't live in São Paulo, though, so it'll be tricky. Thanks, tho. — Kieff | Talk 19:02, 26 April 2008 (UTC)[reply]
Do they have interlibrary loans in Brazil? --Anon, 00:12 UTC, April 27, 2008.

Osmolarity and Cells

This image nicely summarizes the concept in question. Nimur (talk) 14:40, 26 April 2008 (UTC)[reply]

Hi, I'm a beginning student to this field, and I just have a quick question. Actually I'm wondering if my understanding of this is correct.

Imagine you have a cell which you place in a solution. If the solution is: Isotonic = no change in cell size Hypertonic = cells shrink BECAUSE water flows to dilute higher solute concentration outside of cell Hypertonic = cells expand BECAUSE water flows to dilute higher solute concentration inside of cell

Is this correct? It's just that my textbook says that the solutes diffuse and that the water follows them, but this makes no sense (in the hypertonic solution, wouldn't particles flow into the cell, causing it to expand?). Thanks Guycalledryan (talk) 10:33, 26 April 2008 (UTC)[reply]

See our article on tonicity which explains this. The solute will not pass through the cell membrane if the membrane is impermeable to that solute. If the solute cannot be exchanged, the concentration can only then be equalised with an exchange of water. SpinningSpark 12:42, 26 April 2008 (UTC)[reply]
The whole idea is that the cell membrane prevents some particles from diffusing. Due to Entropy, particles concentrated in a small area tend to disperse until the area they can inhabit is homogenized. The deal with cells is that they let some particles cross their membrane (like water) but they don't let others pass through. So, if you put a cell in a concentrated solution (HYPERtonic to the cell) water flows OUT of the cell in an attempt to equalize the concentration outside with the concentration inside, and the cell loses volume (i.e. shrinks). Conversely, if you put a cell in a HYPOtonic solution (like pure H20) water flows into the cell in an attempt to dilute the concentration inside the cell.
The general rule is that systems move towards equilibrium, which in this case is isotonicity. When your book says water follows solute, they're saying that in general the water well go where it can find the highest concentration of solute, because the tendency of the system is to dilute the high concentration. --Shaggorama (talk) 17:34, 26 April 2008 (UTC)[reply]

FUNDAMENTAL ATTRIBUTION ERROR

Am seeking someone who might have participated in a FAE study and /or an individual(s) whom have done some research in this area and has some stimulating theories as to our assumptions that a person's social and enviornmental influences a persons actions vs the kind of individual they are. How often do we find ourselves sliopping int those tendencies to judge?

And are we the Kind of person because of our social and eviornmental influences? ZLAW —Preceding unsigned comment added by Zeldawong (talkcontribs) 14:58, 26 April 2008 (UTC)[reply]

Nuclear Testing on GIs

Can anyone direct me toward a good site that has interviews from GIs used as "guinea pigs" in the Nuclear Test sites during the Cold War. I keep coming up with some site about Guinea pig retinas and medical revolutions. :( —Preceding unsigned comment added by EWHS (talkcontribs) 15:56, 26 April 2008 (UTC)[reply]

The term for this, oddly enough, is Atomic veteran. (Wow, something WP doesn't have an article on! You can even get a license plate with it on it if you were one, in Alabama.) Here's a website all about interviews with veterans who participated in nuclear tests: Atomic Veterans History Project. --Captain Ref Desk (talk) 17:15, 26 April 2008 (UTC)[reply]
Outside of the US, "nuclear test veteran" or "nuclear veteran" seem common Nil Einne (talk) 18:33, 26 April 2008 (UTC)[reply]

Will they ever invent a microwave that wont freak out if I accidently leave my fork in there?

I heard this asked by Adam Corolla and it's always bothered me too. I figure there must be a reason why this happens. Bill Nye said it would be like trying to design a mirror that doesn't reflect. But there are mirrors that dont reflect. Two way mirrors. One side you can see thru. Microwave-safe cans still reflect (though,fainter), and there seems to be no problem there. What is the science here that explains why they cant invent an everything safe microwave?--Sunburned Baby (talk) 21:04, 26 April 2008 (UTC)[reply]

A lot of people misunderstand two-way mirrors -- what they do is reflect some of the light, and let (most of) the rest of it through. Works the same from either direction (hence the name two-way mirror). The apparent directionality comes from the fact that, when one side is lit much brighter than the other, the intensity of the reflection is so much greater than the intensity of the light coming through from the other side, that the eye can't make out the transmitted signal.
However I don't see why a microwave couldn't be developed that would detect that there's a fork in it, and turn itself off so that you don't get the arcing. But you would have to take the fork out before cooking the food. --Trovatore (talk) 21:33, 26 April 2008 (UTC)[reply]
See Two-way mirror. hydnjo talk 23:48, 26 April 2008 (UTC)[reply]
Microwaves are electromagnetic radiation, so they push around electric charges. A water molecule is like a little electric dipole (the oxygen side is negative and the hydrogen side is positive), so when microwaves hit water molecules they move around faster, and this motion of molecules is manifested as heat. Metals are conductors of electricity, which means electric charges can flow through them. When microwaves hit a metal, electric currents flow across the surface of the metal. (It's only the surface and not the whole volume of the metal because the surface currents cancel out the electric field of the microwaves; the waves only penetrate a microscopic distance into the metal.) The problem with putting a fork in the microwave is not simply that it's made of metal, the problem is that it has sharp points. As the electric currents flow across the sharp points, the charge builds up and creates a large electric field; much larger than the field of the microwaves themselves. If the field gets too high (3 kV/mm), the air undergoes electrical breakdown and turns into a plasma, which conducts electricity and makes the microwave oven "freak out".
I actually leave spoons in the microwave all the time, because I understand the physics and know that it's safe. As I said, it's not the metal itself, it's the sharp points act as focal points for the electric field.
So to answer your question, it's impossible to invent a microwave oven that doesn't do this, because it's just the way electromagnetism works. If you have strong electromagnetic waves hitting a sharp metal point, there's going to be a large field there no matter what. —Keenan Pepper 21:41, 26 April 2008 (UTC)[reply]
Understanding these focal points, are there ways to manipulate the arrangement of sharp points so as to prove different mechanics of electrics and electromagnetism? Say, for instance, to put in some forks arranged in a particular formation, along with the traditional ferromagnetic materials organized around a magnet of some kind? 81.93.102.185 (talk) 21:51, 26 April 2008 (UTC)[reply]
It would be far simpler to just use plastic forks. -- HiEv 02:59, 27 April 2008 (UTC)[reply]
I think the solution to this problem would be to create some sort of metal-polymer alloy that is metallic enough to stand up to dishwashers and years of use, but plastic enough not to create sparks in a microwave. Vranak (talk) 22:03, 26 April 2008 (UTC)[reply]
Or maybe the microwave could rearrange its field inside to makd a null at the pointy bits on the fork, so that no sparks are generated, but enough microwaves still heat up the food. You are unlikely to see such a machine though as it would cost more to make. Graeme Bartlett (talk) 22:13, 26 April 2008 (UTC)[reply]
You may not be able to leave a fork in the microwave, but how about making one along with cooking your dinner? (See http://www.nature.com/nature/journal/v399/n6737/full/399668a0.html or http://www.sciencedaily.com/releases/1999/06/990622055733.htm) It would make your TV dinner rather costly and wasteful, though.--Lisa4edit (talk) 02:14, 27 April 2008 (UTC)[reply]

Moment of Inertia

I was given a moment of inertia problem that went somewhat at follows. We are considering a trebuchet, with a 60kg mass .14meters from the axel, and a .12kg rock 2.86meters from the axel. We are then asked to find the max speed of the rock. The question itself I answered, getting something close to 25m/s, and I did it using conservation of energy. But I have a few questions. First, is there another method of answering the question? Second, when doing conservation of energy, would the potential energy be mgh? Finally, if we wanted to determine whether the big or the small object fell, would we see which mass times distance was greater, or would we calculate which moment of inertia was greater, and why? —Preceding unsigned comment added by 76.69.240.138 (talk) 21:30, 26 April 2008 (UTC)[reply]

I remembered a TV program a few years back (History channel I think). They built a trebuchet and found that the wheels were not just for rolling the siege engine into position, but also figured into the equations of firing the missle. I tried to find a link for you, but I only found s.o. else who had seen the program mentioning it in a physics forum. Our page says the fact is disputed among scientist, but unfortunately doesn't have a reference link. --Lisa4edit (talk) 03:02, 27 April 2008 (UTC)[reply]
I saw that program, too. I thought the conclusion was that the wheels are needed for the recoil, to prevent forces from building up in the frame which would otherwise shake it apart. StuRat (talk) 06:00, 27 April 2008 (UTC)[reply]
I think I know what program you're talking about. I remember seeing it on Discovery Channel. Two competing groups built an English (fixed weight) and a French (hanging weight) trebuchet. It was pretty cool, but I couldn't find its name last time I checked. — Kieff | Talk 11:50, 27 April 2008 (UTC)[reply]
Conservation of energy is a good approach. Potential energy lost by counterweight - which is mgh - is equal to potential energy gained by rock and arm plus kinetic energy gained by rock, arm and counterweight. You could work out the net torque on the trebuchet arm at each point in its motion, then divide by the moment of inertia of the arm, rock and counterweight to find the angular acceleration of the system, then integrate this to work out the angular velocity and hence linear velocity of the rock at point of release - but you should get the same answer.
Calculating the net torque (which is how you know that the counterweight is heavy enough to move arm and rock) just involves multiplying weight x distance from pivot for rock, arm and counterweight - if the arm is a uniform beam then it can be treated as a point weight at its geometric centre. Don't confuse torque with momement of intertia - that is like confusing force and mass.
Note that both approaches neglect energy lost due to friction, air resistance, deformation of the terbuchet frame, and linear motion of the trebuchet if it is on wheels - so in real life your trebuchet performance will not be as good as your calculation suggests. You are also assuming that the trebuchet is a simple lever. More sophisticated trebuchets - see right - had the rock in a sling and the counterweight in a pivoted cage, so they were not just a simple lever. The mathematical anaylsis of a sling trebuchet is much more difficult, because of the complications introduced by the rotations of rock and counterweight about the additional pivots. Gandalf61 (talk) 08:32, 27 April 2008 (UTC)[reply]

April 27

Flower name

Does anyone know the name of this flower? I don't know what it is and I would like to add this image to the appropriate article. --AutoGyro (talk) 02:31, 27 April 2008 (UTC)[reply]
What's my name?

Looks like a Narcissus (a.k.a. a "daffodil") of some sort to me. -- HiEv 02:50, 27 April 2008 (UTC)[reply]

Concur with daffodil. But as there are thousands of different cultivars it would take narcissophile of some expertise to put a varietal name to it. Richard Avery (talk) 11:55, 27 April 2008 (UTC)[reply]
This is almost certainly the most commonly grown daffodil: King Alfred [24].--Eriastrum (talk) 15:04, 27 April 2008 (UTC)[reply]

Hampster who injured his mouth with too long teeth

My name is Popeye. I am a Teddy Bear Hamster. I have been in misery for the last 7 hours, I haven't left my bed and I am making choking noises. I cut the roof of my mouth with my very long teeth. My master took me to our friend at the pet store who cut off about 1 millimeter of my bottom teeth. But I am still making the choking noise and I won't leave my bed. My master is afraid I am going to die. Please help----Popeye, I am about 6 months old. —Preceding unsigned comment added by 68.164.84.9 (talk) 03:11, 27 April 2008 (UTC)[reply]

Popeye's master should have given Popeye some things to gnaw on that wear down teeth. See Tooth (animal)#Rodent. I am not sure what can be done now except wait for the damage to heal, if possible. If I were Popeye's master, I would immediately call a vet and ask for advice. The vet may have some liquid food to drop in Popeye's mouth to feed him while he recovers. =Axlq 04:18, 27 April 2008 (UTC)[reply]

Quantum: Potential between the Electrons

Suppose there are two electrons in an one-dimentional spatial space (imaging two electrons in the same wire of infinite length). And the wave function of two electrons is where and are the postions of the respective electron. Then should be the probability density function for finding the first electron at and the second electron at . And the normalization condition of the wave function should be

(1)

And the potential of the electron should be

(2)

according to Coulomb's law and the fact that . Where is the distance between the two electrons. For two electrons in one-dimensional space, it should be

(3)

Replace (3) into (2), the potential becomes

(4)

Because of the Uncertainty Principle, we can not make sure the position of each electron. But we know the probability of finding them by . (one way to find out the potential of the electron is to introduce quantum electrodynamics according to Gandalf61's suggestion in the discussion Determine the Force between the Electrons which, however, seems too complex to me) Then my idea to find out the effective potential in discrete form is simply to calculate the weighted sum

where , , ,... are probabilities of finding the two electrons at different positions and , , ,... are the potentials at the corresponding positions. So

where is the probability of finding the first electron at and the second electron at . Or, the effective potential in continuous form

and my question is can I determine the potential of the two electrons by the equation ? - Justin545 (talk) 03:16, 27 April 2008 (UTC)[reply]

I don't mean to be disrespectful or denegrating, but honestly, this looks suspiciously like a homework problem to me. =Axlq 04:12, 27 April 2008 (UTC)[reply]
It appears they've done prior work, indicating that they require assistance on something researched. Wisdom89 (T / C) 04:16, 27 April 2008 (UTC)[reply]
I am neither doing a homework nor writing a paper. Studying things about quantum is just one of my hobbies while I am free. - Justin545 (talk) 05:15, 27 April 2008 (UTC)[reply]
You have the right form for the expectation value of the potential between the electrons, however, you have to use in solving the Schrodinger equation (written below) by which you determine the wave function. Until you solve the Schrondinger equation you won't know the form of Ψ and hence you couldn't compute the expectation value anyway.
Here I've included the explicit time dependence because your proposed boundary condition (two electron on an infinite wire) would not admit any non-trivial time independent solutions. Dragons flight (talk) 08:30, 27 April 2008 (UTC)[reply]
Does that mean I can know the form of by solving the following Schrondinger
? Thanks. - Justin545 (talk) 09:43, 27 April 2008 (UTC)[reply]
Yes, that looks correct to me. If you extend this to three spatial dimensions, you have the hydrogen-like atom, which is perhaps the most useful exactly solvable model in quantum mechanics. Make sure to work in the center of momentum frame and use the reduced mass; then the problem separates nicely into a center-of-mass motion part (whose eigenstates are plane waves), and a relative motion part (whose eigenstates are atomic orbitals). —Keenan Pepper 18:04, 27 April 2008 (UTC)[reply]

Name of material

Does anyone know what kind of material is this (the uniform)? Is it the same material worn by professional StarCraft players on competitions? Looks similar at least --217.14.103.168 (talk) 07:38, 27 April 2008 (UTC)[reply]

The uniform would be called a "flight suit", but that won't get you much closer to the material I'm afraid.--Lisa4edit (talk) 09:49, 27 April 2008 (UTC)[reply]
It looks to me like the sort of suit that Formula One drivers wear and is probably a composite of several layers, the key one being Nomex for fire resistance. This site has some information. SpinningSpark 10:54, 27 April 2008 (UTC)[reply]
I found this mail address: office@stevefossett.com They might be able to help you. The racing suit family looks likely, because "Marathon Racing Inc." runs the site. That suit had some extra "waste product disposal" features though. I don't think that's standard for racing suits. He wore a fire resistant suit during his glider flight, but there's no mention of this one except that it was silver. --Lisa4edit (talk) 11:01, 27 April 2008 (UTC)[reply]
BTW, I've never seen Starcraft players but I'm somewhat doubtful they usually wear anything like that. I've looked at a few pictures including the one here StarCraft professional competition and although none of them are that good, I would suspect it's more likely some kind of simple & cheap somewhat shiny synthethic, perhaps Lycra Nil Einne (talk) 19:45, 27 April 2008 (UTC)[reply]

Cyclic carbonates ?

In this article (http://www.sciencedaily.com/releases/2008/04/080424103217.htm) they mention cyclic carbonates. Wikipedia doesn't seem to have an article on them and my chemistry is a couple of decades old. I seem to have a faint memory though that says most of them are toxic?? --Lisa4edit (talk) 09:03, 27 April 2008 (UTC)[reply]

We have an article on carbonates and individual cyclic carbonates such as ethylene carbonate but apparently no article for them as a group. Here is the Material Safety Data Sheet for ethylene carbonate. Not very pleasant stuff. SpinningSpark 11:22, 27 April 2008 (UTC)[reply]
Also, the article Carbonate ester names a few more of them (individual articles as well), but without actually calling them cyclic carbonates. SpinningSpark 11:27, 27 April 2008 (UTC)[reply]

Thanks. So the murky old memory wasn't that far off. Not really the stuff to look forward to having in the neighborhood. Hope they'll take adequate safety precautions. --Lisa4edit (talk) 11:54, 27 April 2008 (UTC)[reply]

Fortunately even the simplest cyclic carbonate is a liquid (ethylene carbonate, bp 260 °C), so it's not going to create a cloud of toxic gas if there's a leak from a storage tank. DMacks (talk) 18:59, 27 April 2008 (UTC)[reply]

medical

how to control trigylcides —Preceding unsigned comment added by 60.243.25.44 (talk) 09:13, 27 April 2008 (UTC)[reply]

Presumably you mean triglyceride. If so hypertriglyceridemia discusses the medical aspects of excessive levels of triglycerides in the blood. -- Alan Liefting (talk) - 11:02, 27 April 2008 (UTC)[reply]
This may be of interest to you: Hypolipidemic agent. Also see Statins and fibrates. Wisdom89 (T / C) 18:12, 27 April 2008 (UTC)[reply]

Humans and ability of interactions with others

There is a law that says humans are only capable of dealing with about 150 people. WP has a page on it. Any ideas? -- Alan Liefting (talk) - 09:36, 27 April 2008 (UTC)[reply]

Never heard of that one before. Could you give us a clue or link where you've encountered that. I checked the "social interaction" and "Interpersonal Interaction" pages briefly, but nothing stuck out, except that they still seem to need a lot of work. --Lisa4edit (talk) 09:48, 27 April 2008 (UTC)[reply]

It's called somebody's number and it's extrapolated from the size of a part of our brains (the cerebral cortex?) relative to other primates. I have read the article here; I will try to remember the person's name. moink (talk) 11:23, 27 April 2008 (UTC)[reply]
Got it. Dunbar's number. moink (talk) 11:29, 27 April 2008 (UTC)[reply]
Yep, I had also just found it from what you said, but it is a hypothesis at first glance and the 150 are an estimate. Worth further reading though. --Lisa4edit (talk) 11:40, 27 April 2008 (UTC)[reply]

Why is frozen milk yellow?

Why is frozen milk yellow?--Shantavira|feed me 11:12, 27 April 2008 (UTC)[reply]

I've been dealing quite a bit with frozen milk of late, though not cow's milk. I don't find that the frozen milk is any different colour from the liquid milk. It does have a slight yellowish cast, either frozen or liquid, which is true of any "whole" (full-fat) milk. Skim (non-fat) milk often seems bluish. moink (talk) 11:21, 27 April 2008 (UTC)[reply]
The fat in the milk freezes at a lower temperature than the other watery part and rises to the top when that part freezes (compare ice from the freezer to ice cream). According to milk, the yellow colour comes from fat globules. Skim milk won't turn yellow, according to this (which isn't very encyclopedic, I admit).
Zain Ebrahim (talk) 11:26, 27 April 2008 (UTC)[reply]
Ah, I have been freezing milk in transparent containers and looking at it from the side, so I see all the milk. Shantavira, have you been looking at your frozen milk from the top? (thus seeing the cream?) moink (talk) 11:34, 27 April 2008 (UTC)[reply]
Liquid milk appears white not because it is white but because the particles (fat, protein) scatter the light equally across the spectrum. My guess is that when you freeze the milk, you see the actual colour of the milk (the fat part will probably be yellow).--Shniken1 (talk) 12:04, 27 April 2008 (UTC)[reply]
I believe it is fat, which, for some reason, finds it's way to the outside of the container, not just the top. The freezing process causes the fat to seperate, and fat is lighter than milk, but it doesn't seem to exclusively rise to the top until the ice thaws, but rather it sticks to the container. I would guess that this is because the density of frozen milk (being mostly water) is less, so that fat is no longer significantly lighter. Why it sticks to the inside of the container must have something to do with relative surface tensions. StuRat (talk) 15:46, 27 April 2008 (UTC)[reply]
Milk is an emulsion. That makes the process of freezing a whole lot more complex. You have a transport problem where part of the material follows fluid dynamics while the other is solid. I have no reference, but the observed separation of fat might be similar to frost heaving. With the fat behaving like the soil. Whether the fat is yellow or more whitish depends to some degree on what the cow ate (see here http://jds.fass.org/cgi/reprint/7/2/146.pdf?ck=nck). See one can turn something simple like frozen milk into something rather complex. Thanks for asking this question. I had fun looking for answers. ;-) Lisa4edit (talk) 17:17, 27 April 2008 (UTC)[reply]
Keep in mind that milk, being a natural product, varies slightly in color from place to place and location to location. Something as simple as breed of cow which produced the milk, or the type of food fed to the cow (e.g. silage versus pasturage) can change the color and/or flavor of the milk. Color can vary based on the amount of carotenes in the cows' diets. You tend to see differences happening in the Northern Hemisphere at about this time, as cows which have been kept inside all winter (and fed silage) make a transition outside to graze on pasture. -- 128.104.112.85 (talk) 17:24, 27 April 2008 (UTC)[reply]

Spherical coordinates

Hi, If the motion of a body is traced in cartesian and spherical coordinate systems, what is the basic difference, other than the method of representing the points?? Will the shape differ?? 116.68.71.144 (talk) 14:06, 27 April 2008 (UTC)A 15 year old.[reply]

The shape wouldn't differ. The mathematical equations describing them would be different. Which coordinate system gives you the easier equation depends on the motion you want to describe. --Lisa4edit (talk) 15:03, 27 April 2008 (UTC)[reply]
Agreed, any path can be represented in spherical, cylindrical, or cartesian coords. However, since it's far easier to show straight line motion with cartesian coords and far simpler to represent near-circular motion, like orbits, in spherical coords, which system is chosen to represent the motion may tell you something about the expected motion. StuRat (talk) 15:23, 27 April 2008 (UTC)[reply]

Condition name

Is there a name for a condition where people can not remember the name of an object such as a fruit? For instance they me be able to remember the name of a strawberry because the eat strawberry jam almost every day but can not remember the name of a mango after a few weeks or months when handed the fruit, calling it a guava? 71.100.11.39 (talk) 14:09, 27 April 2008 (UTC) [reply]

Aphasia seems to be what you're looking for. At least one type of it. --Lisa4edit (talk) 14:21, 27 April 2008 (UTC)[reply]
I would have answered, but I can never recall the name... :-) StuRat (talk) 15:16, 27 April 2008 (UTC)[reply]
There is also agnosia, where a patient fails to recognise an object and therefore can not apply a name to it. A strawberry would be perceived as a roundish red object with small pimply kernels and some green squashed bits on one end, but you would have no idea about its use.
Forgetting little used terms is quite common in a variety of neurological disorders (dementia or Alzheimer´s). Actually, It is quite normal for anybody to confuse terms which are infrequently used. I am sure that I would mix up terms for tropical fruit, different types of pasta and the names of various names of sausages if there was not a label in front of it. --Cookatoo.ergo.ZooM (talk) 16:23, 27 April 2008 (UTC)[reply]
Knew there was another one that I had forgotten ;-) The interesting thing is that they found that the brain seems to store stuff by category. That's why you are more likely to name a vegetable with a wrong vegetable name rather than that for a sausage. You might still do that if the sausage name has a similar "sound pattern" or you ate it at the same time you ate that fruit.--Lisa4edit (talk) 17:32, 27 April 2008 (UTC)[reply]

dinasours

how dinasours relates to the environment —Preceding unsigned comment added by 70.23.173.57 (talk) 18:05, 27 April 2008 (UTC)[reply]

I'm a little perplexed as to what your question means..could you elaborate pleas? Wisdom89 (T / C) 18:09, 27 April 2008 (UTC)[reply]
Our Dinosaur article mentions a couple of factors:
However, by the late Cretaceous, the environment was changing dramatically and
...no closely related species exist to provide zygotes or a suitable environment... --hydnjo talk 18:30, 27 April 2008 (UTC)[reply]

A few questions on drugs

I'm filling out a general health test, completely open-book, open-note, open-person (talk to anybody), and I've done just about all of it. I have a few questions though:

  • What is it called when alcohol is consumed faster than it can be processed by the body?
  • The use of alcohol with GHB can produce so much sedation that your brain stops telling you to — ? (think/eat/breathe/move)
  • As prescription drug — has increased with direct-to-consumer advertising, there has been a parallel increase in prescription drug use.
  • — cannot write prescriptions for themselves, but they do have considerable influence over what physicians do.
  • Some people are using GHB every two to four hours because if they don't use it, they cannot sleep, and get extremely anxious to the point of becoming —.
  • As the brain adapts to the hyper-release of dopamine with — drug use, lack of the drugs causes severe craving and addiction.

The last four are fill-in-the-blanks and seem rather vague to me to narrow it down, but then again, I don't know too much about drugs. Thanks in advance for your help! --Fbv65edeltc // 19:58, 27 April 2008 (UTC)[reply]

Poteries

Peut-on enregistrer les bruits aux potteries? —Preceding unsigned comment added by 99.237.101.48 (talk) 20:21, 27 April 2008 (UTC)[reply]

Est-ce que c'est une plaisanterie? What noise? And I guess the translator got "potteries" wrong. --Lisa4edit (talk) 21:02, 27 April 2008 (UTC)[reply]

Well, there was that X-Files episode where Jesus' command to Lazarus got recorded on a pot being spun in the vicinity, and when it was played as a record, it brought dead people back to life (or at least undeath, which seems a bit odd). --Trovatore (talk) 21:08, 27 April 2008 (UTC)[reply]
C'est "poteries", pas "potteries". Trovatore: Pourquoi penses-tu que c'est une plaisanterie?
Est-ce que l'épisode est réaliste? Merci. --99.237.101.48 (talk) 21:41, 27 April 2008 (UTC)[reply]

A drug called APO 300

My 86 year old forther is taking a small blueish coloured pill with the letters APO 300 on one side. I would like to know what this drug is, and what it is for. He takes it at lunch hour

Thanks
Tom —Preceding unsigned comment added by 24.36.232.146 (talk) 20:22, 27 April 2008 (UTC)[reply]

The drug is called Apo-ranitidine (unfortunately, we have no article on this drug) and serves to reduce the production of stomach acid. This drug can be used in the treatment of various conditions, including heartburn, gastroesophageal reflux disease, and Zollinger-Ellison syndrome. It's intended purpose is usually to reduce pain from these conditions, although it can also prevent ulcers in some circumstances. It is from the general family of drugs called H2 antagonist. You can find more information about the drug here. Someguy1221 (talk) 21:16, 27 April 2008 (UTC)[reply]

Ranitidine, commercially known as Zantac. -- Kesh (talk) 21:25, 27 April 2008 (UTC)[reply]

Norepinephrine

It says that norepinephrine is both a stress hormone and a neurotransmitter. Does that mean when you are really stressed out you can think better, because you have higher levels of norepinephrine, or something like that? --131.215.166.126 (talk) 21:34, 27 April 2008 (UTC)[reply]

It means that norepenphrine functions and behaves both as a hormone (circulates in the blood) and a neurotransmitter (communication between neurons). It is secreted from the adrenal medulla along with epinephrine and trace amounts of dopamine, in addition to being present in the central nervous system that acts as a stimulatory neurotransmitter. Wisdom89 (T / C) 21:36, 27 April 2008 (UTC)[reply]
And yes, in a manner of speaking, norepinephrine does improve cognition as it is essential for maintaining alert and wakefulness, in addition to a slew of other functions. Wisdom89 (T / C) 21:37, 27 April 2008 (UTC)[reply]